Functional: Female SUI: Difference between revisions

From UrologySchool.com
Jump to navigation Jump to search
No edit summary
 
(57 intermediate revisions by the same user not shown)
Line 7: Line 7:
*<span style="color:#ff0000">'''Urgency urinary incontinence (UUI): symptom of urinary leakage that occurs in conjunction with the feeling of urgency and a sudden desire to urinate that cannot be deferred.'''</span>
*<span style="color:#ff0000">'''Urgency urinary incontinence (UUI): symptom of urinary leakage that occurs in conjunction with the feeling of urgency and a sudden desire to urinate that cannot be deferred.'''</span>
*<span style="color:#ff0000">'''Mixed urinary incontinence refers to a combination of SUI and UUI'''</span>
*<span style="color:#ff0000">'''Mixed urinary incontinence refers to a combination of SUI and UUI'''</span>
*<span style="color:#ff0000">'''Mesh located in the lower urinary tract is termed a ''perforation,'' and extrusion of mesh through the skin or vagina is termed ''exposure'''''</span>
==Epidemiology ==
==Epidemiology ==


Line 24: Line 25:
*** '''ALPP > 90 cm H2O signifies no or very little ISD'''
*** '''ALPP > 90 cm H2O signifies no or very little ISD'''


== Preoperative Assessment ==
== Differential Diagnosis of SUI ==
*'''<span style="color:#ff0000">Options (7)</span>'''
*#'''<span style="color:#ff0000">Overflow incontinence</span>'''
*#'''<span style="color:#ff0000">Detrusor overactivity incontinence</span>'''
*#'''<span style="color:#ff0000">Low bladder compliance</span>'''
*#'''<span style="color:#ff0000">Stress-induced detrusor overactivity</span>'''
*#'''<span style="color:#ff0000">Diverticulum</span>'''
*#'''<span style="color:#ff0000">Urinary fistula</span>'''
*#'''<span style="color:#ff0000">Ectopic ureter</span>'''
==Diagnosis and Evaluation of Patients Wanting Surgery for SUI ==
===Recommended Investigations===
====Mandatory (4):====
#'''<span style="color:#ff0000">History (including assessment of bother) and Physical Exam</span>'''
#'''<span style="color:#ff0000">Objective demonstration of stress urinary incontinence</span>'''
#'''<span style="color:#ff0000">Urinalysis</span>'''
#'''<span style="color:#ff0000">PVR</span>'''
=== History and Physical Exam===


* '''As per 2017 AUA Female SUI Guidelines, mandatory work-up of SUI includes (4):'''
==== History ====
*# '''History (including assessment of bother) and physical exam'''
*'''<span style="color:#ff0000">Characterize incontinence</span>''' (stress, urgency, mixed, continuous, without sensory awareness)
*#* Although slings are an effective treatment for both genuine SUI and mixed urinary incontinence (MUI), a thorough understanding of preoperative symptomatology will help guide counseling and treatment decisions; '''pre-operative''' '''urgency is associated with worse outcomes after sling surgery'''
**Presence of coughing, sneezing, lifting, walking, or running as initiators of incontinence increases the likelihood of SUI as the cause of urinary leakage
*# '''Objective demonstration of SUI'''
**'''<span style="color:#ff0000">Chronicity of symptoms</span>'''
*# '''Urinalysis'''
**'''<span style="color:#ff0000">Frequency, bother, and severity</span>''' of incontinence episodes. Pad or protection use.
*# '''PVR'''
***'''<span style="color:#ff0000">An assessment of bother is paramount to the decision to operate in the index patient.</span>'''
* '''Urodynamic studies are not needed in all patients before SUI treatment'''
* '''<span style="color:#ff0000">Associated</span>'''
** '''Indications for urodynamics in SUI (9):'''
**'''<span style="color:#ff0000">Urinary tract symptoms</span>''' (e.g., urgency, frequency, nocturia, dysuria, hematuria, slow flow, hesitancy, incomplete emptying)
**# '''Diagnosis of SUI remains unclear'''
**'''<span style="color:#ff0000">Pelvic symptoms</span>''' (e.g., pelvic pain, pressure, bulging, dyspareunia)
**# '''Negative stress test'''
**'''<span style="color:#ff0000">GI symptoms</span>''' (e.g., constipation, diarrhea, splinting to defecate)
**# '''Concomitant overactive bladder (OAB) symptoms'''
*'''<span style="color:#ff0000">Menopausal status</span>'''
**# '''Possibility of neurogenic bladder'''
*'''<span style="color:#ff0000">Obstetric history</span>''' (e.g., gravity, parity, method of delivery)
**# '''Evidence of dysfunctional voiding'''
*'''<span style="color:#ff0000">Previous pelvic surgeries</span>'''
**# '''Elevated PVR'''
*'''<span style="color:#ff0000">Past medical history</span>''' (e.g., hypertension, diabetes, history of pelvic radiation)
**# '''Grade ≥3 prolapse'''
*'''<span style="color:#ff0000">Current and past medications</span>'''
**# '''Prior lower urinary tract surgery'''
*'''<span style="color:#ff0000">Fluid, alcohol, and caffeine intake</span>'''
**# '''An unexplained abnormality on urinalysis'''
*'''<span style="color:#ff0000">Previous treatments for incontinence</span>''' (e.g., behavioral therapy, Kegel exercises/pelvic floor muscle training, pharmacotherapy, surgery)
*** '''If significant prolapse is present, UDS should be performed with and without a pessary'''
*'''<span style="color:#ff0000">Patient’s expectations of treatment</span>''' (patient-centered goals)
*** '''In patients with DO,''' '''treatment options other than sling surgery should be considered''' because stress-induced DO may be difficult to treat with a sling alone.
* '''<span style="color:#ff0000">History alone, while helpful, does not definitively diagnose SUI in women</span>'''
*** Abnormally small bladder capacity and decreased compliance may also negatively affect the outcomes of sling surgery, and these factors should also be considered.
====Physical exam (6): ====
#'''<span style="color:#ff0000">Stress test (supine and/or standing) with comfortably full bladder</span>'''
#'''<span style="color:#ff0000">Focused abdominal examination</span>'''
#'''<span style="color:#ff0000">Urethral mobility</span>''' (Q-tip test or other method)
# '''<span style="color:#ff0000">Pelvic prolapse</span>''' (any method)
#'''<span style="color:#ff0000">Vaginal atrophy/estrogenization status</span>'''
#'''<span style="color:#ff0000">Focused neurologic examination</span>'''
====Objective demonstration of SUI====
*'''<span style="color:#ff0000">Stress test (supine and/or standing) with comfortably full bladder</span>'''
** Considered positive if involuntary urine loss from the urethral meatus is witnessed coincident with increased abdominal pressure
***A positive stress test had a high sensitivity and specificity for detecting SUI
**'''If leakage is not witnessed in the supine position, the test may be repeated in the standing position to facilitate the diagnosis'''
====Urinalysis====
*Screen for abnormalities (microscopic hematuria, pyuria, etc.), which may prompt further investigations and reveal underlying cause (bladder tumour, for example) of incontinence
====Post-void Residual (PVR)====
*May prompt further investigations and reveal underlying cause (overflow, for example) of incontinence
===Other Tests ===
===== Questionnaires=====
*Overall, low strength of evidence due to limited number of studies for each questionnaire
=====Q-tip test=====
* '''Positive test is unlikely to aid in the diagnosis of SUI; SUI may exist without urethral hypermobility and vice versa.'''
*Can provide potentially useful information regarding the degree of urethral mobility
===== Pad test=====
*May confirm the presence of incontinence but does not distinguish the specific type
===Additional Evaluations===
*'''<span style="color:#ff0000">Should be considered for patients with the following conditions (8)</span>''':
*#'''<span style="color:#ff0000">Neurogenic lower urinary tract dysfunction</span>''' (known or suspected)
*#'''<span style="color:#ff0000">Inability to demonstrate stress urinary incontinence</span>'''
*#'''<span style="color:#ff0000">Inability to make definitive diagnosis based on symptoms and initial evaluation</span>'''
*#'''<span style="color:#ff0000">Elevated post-void residual</span>''' per clinician judgment
*#'''<span style="color:#ff0000">Evidence of significant voiding dysfunction</span>'''
*#'''<span style="color:#ff0000">Urgency-predominant mixed urinary incontinence</span>'''
*#'''<span style="color:#ff0000">Abnormal urinalysis,</span>''' such as unexplained hematuria or pyuria
*#'''<span style="color:#ff0000">High grade pelvic organ prolapse</span>''' (POP-Q stage 3 or higher) '''if SUI not demonstrated by pelvic organ prolapse reduction'''
*'''May be performed in patients with:'''**Concomitant overactive bladder symptoms
**Failure of prior anti-incontinence surgery
**Prior pelvic prolapse surgery
====Cystoscopy====
*'''<span style="color:#ff0000">Should not be performed in index patients for the evaluation of SUI</span>'''
*'''<span style="color:#ff0000">Indications (3):</span>'''
*#'''<span style="color:#ff0000">Suspected bladder pathology</span>''' based on history or concerning findings on physical exam or urinalysis (e.g. microhematuria)
*#'''<span style="color:#ff0000">Structural lower urinary tract abnormality</span>'''
*#'''<span style="color:#ff0000">Patients undergoing certain surgical procedures (e.g., midurethral (MUS) or pubovaginal fascial (PVS) slings)</span> to confirm the integrity of the lower urinary tract and the absence of foreign body'''  
*#'''<span style="color:#ff0000">History of prior anti-incontinence surgery or pelvic floor reconstruction, particularly if mesh or suture perforation is suspected</span>'''
*#*'''Perforation should be suspected with new onset of lower urinary tract symptoms, hematuria, or recurrent UTI'''
==== UDS====
*'''<span style="color:#ff0000">May be omitted for the index patient desiring treatment when SUI is clearly demonstrated'''
**'''<span style="color:#ff00ff">VALUE (NEJM 2012)</span>'''
***Population: 630 females with uncomplicated SUI
***Randomized to preoperative office evaluation and urodynamic tests vs. evaluation only
***Primary outcome: Treatment success at 12 months, defined as a reduction in the score on the Urogenital Distress Inventory of 70% or more and a response of “much better” or “very much better” on the Patient Global Impression of Improvement
***Results: preoperative office evaluation alone was not inferior to evaluation with urodynamic testing
***[https://pubmed.ncbi.nlm.nih.gov/22551104/ Nager, Charles W., et al. "A randomized trial of urodynamic testing before stress-incontinence surgery." ''New England Journal of Medicine'' 366.21 (2012): 1987-1997.]
*'''<span style="color:#ff0000">May be performed at the urologist’s discretion in certain non-index patients, including but not limited to (8):'''
*#'''<span style="color:#ff0000">Neurogenic lower urinary tract dysfunction</span>'''
*# '''<span style="color:#ff0000">Unconfirmed SUI</span>'''
*#'''<span style="color:#ff0000">Mismatch between subjective and objective measures</span>'''
*#'''<span style="color:#ff0000">Elevated PVR per clinician judgment</span>'''
*#'''<span style="color:#ff0000">Significant voiding dysfunction</span>'''
*# '''<span style="color:#ff0000">Significant urgency, UUI, overactive bladder (OAB)</span>'''
*#'''<span style="color:#ff0000">History of prior pelvic organ prolapse surgery</span>'''
*#'''<span style="color:#ff0000">History of prior anti-incontinence surgery</span>'''
* '''If significant prolapse is present, UDS should be performed with and without a pessary'''
* '''In patients with DO,''' '''treatment options other than sling surgery should be considered''' because stress-induced DO may be difficult to treat with a sling alone.
* Abnormally small bladder capacity and decreased compliance may also negatively affect the outcomes of sling surgery, and these factors should also be considered.
== Management ==


== Alternative Treatment Options ==
=== Options ===
*'''<span style="color:#ff0000">Observation</span>'''
*'''<span style="color:#ff0000">Non-surgical interventions (4):</span>'''
*#'''<span style="color:#ff0000">Urethral plugs</span>'''
*#'''<span style="color:#ff0000">Vaginal inserts</span>'''
*#'''<span style="color:#ff0000">Continence pessary</span>'''
*#'''<span style="color:#ff0000">Pelvic floor muscle training (± biofeedback)</span>'''
*'''<span style="color:#ff0000">Surgical intervention</span>'''
**'''<span style="color:#ff0000">Options (4):</span>'''
**#'''<span style="color:#ff0000">Periurethral bulking agents</span>'''
**# '''<span style="color:#ff0000">Midurethral sling (MUS)</span>'''
**#'''<span style="color:#ff0000">Autologous pubovaginal sling (PVS)</span>'''
**#'''<span style="color:#ff0000">Burch culposuspension</span>'''
*Physicians should not offer stem cell therapy for SUI outside of investigative protocols
====Observation====
=====Indications (2)=====
#'''<span style="color:#ff0000">Patient expresses minimal subjective bother due to the SUI</span>'''
#*The degree of bother should be considered when considering treatment
#'''Not candidate for other forms of therapy'''
==== Non-surgical interventions====


* '''<span style="color:#ff0000">Options (4):'''
#'''<span style="color:#ff0000">Urethral plugs</span>'''
# '''<span style="color:#ff0000">Vaginal inserts</span>'''
#'''<span style="color:#ff0000">Continence pessary</span>'''
#'''<span style="color:#ff0000">Pelvic floor muscle training (± biofeedback)</span>'''
===== Pelvic floor physical therapy =====
* Patient must be willing and able to commit to regularly and consistently performing pelvic floor training for this to be successful.
====Surgical intervention====
* In most cases of incontinence, surgery should not be considered until more conservative management has failed.
* In most cases of incontinence, surgery should not be considered until more conservative management has failed.
** '''Initial conservative therapy''' includes:
*** Patient self-awareness and education
*** Dietary modification
*** Fluid restriction
*** Weight loss
*** Pelvic floor muscle training
* The submucosal injection of '''periurethral bulking agents''' through a cystoscope into the urethra is a minimally invasive, mildly successful treatment for SUI.


== Pubovaginal Slings (PVS) ==
* '''<span style="color:#ff0000">Options (4):'''
#'''<span style="color:#ff0000">Bulking agents</span>'''
#'''<span style="color:#ff0000">Synthetic midurethral slings (MUS)</span>'''
#'''<span style="color:#ff0000">Autologous pubovaginal sling (PVS)</span>'''
#'''<span style="color:#ff0000">Burch culposuspension</span>'''
=====Bulking agents =====
*Trade name: Bulkamid
**FDA approved in 2020[https://www.accessdata.fda.gov/cdrh_docs/pdf17/P170023B.pdf §]
*'''Little long-term data'''
*'''<span style="color:#ff0000">Indications (2):</span>'''
*#'''<span style="color:#ff0000">Patients who wish to avoid more invasive surgical management or are concerned with the lengthier recovery time after surgery</span>'''
*#'''<span style="color:#ff0000">Patients who experience insufficient improvement following a previous anti-incontinence procedure.</span>'''
*'''<span style="color:#ff0000">Disadvantage (1)</span>'''
*#'''<span style="color:#ff0000">Patients should be counseled on the expected need for repeat injections</span>'''
*Adverse events[https://www.accessdata.fda.gov/cdrh_docs/pdf17/P170023C.pdf §]
**Pain at implantation (13%)
**Acute urinary retention (6%)
**Urinary tract infection (4%)
**Blood in urine (2%)


* '''Indicated for treatment of incontinence associated with a deficiency in a portion of the midurethral complex, hypermobility, ISD, MUI, concomitant cystoceles, urethral diverticula, and neurologic conditions'''
===== Midurethral Slings (MUS) =====
* '''Anatomy and Mechanics of a PVS'''
*Most studied surgical treatment for female SUI
** '''Positioned at the bladder neck to provide urethral compression without obstruction during times of increased intraabdominal pressure.'''
*Other than bulking agents, MUS is also the least invasive surgical options to treat SUI
** The current concept of PVS comes from using a shorter free graft of rectus fascia whose tension could be adjusted
* '''PVS Sling Materials'''
** '''Autologous, allograft, xenograft, and synthetic materials have been used for the construction of a PVS'''
** Although there is complete biocompatibility of the autologous sling and negligible urethral perforation, biologic graft and synthetic prosthetic materials have been increasingly used to decrease operative time, morbidity, pain, and hospital stay
** '''Autologous fascia PVS'''
*** '''Gold standard for management of ALL forms of SUI'''
*** '''Advantages compared to alternative sling materials (2):'''
***# '''Minimal tissue inflammation'''
***# '''Negligible risk of urethral erosion'''
*** '''Disadvantages compared to alternative sling materials (2):'''
**** '''Increased operative time, hospital stay, postoperative pain, risk of suprapubic wound seroma, and risk of incisional hernia'''
*** '''Most commonly used autologous materials (2):'''
***# '''Rectus abdominis fascia harvested from the abdominal wall'''
***#* '''Most commonly used'''
***# '''Fascia lata harvested from the lateral thigh'''
***#* '''Fascia lata is the preferred autologous material for PVSs in patients with a history of prior ventral hernia repair;''' unlike rectus facia''',''' the recovery time is less and there is no risk of future abdominal hernia formation.
***#* Disadvantages of fascia lata compared to rectus abdominis:
***#*# Requires repositioning of the patient
***#*# Increased operative time
***#*# Operating in an area unfamiliar to most pelvic surgeons
***# '''Improvement of SUI is similar with rectus abdominis fascia or fascia lata PVS'''
** '''Allograft PVS'''
*** Were introduced in an effort to reduce overall morbidity, operative time, and pain related to graft procurement
*** Currently derived from either cadaveric fascia lata or acellular human dermis
*** Allografts from cadavers raise the concern of potentially transmitting illnesses such as HIV, hepatitis, and Creutzfeldt-Jakob prion disease
**** The estimated risk of HIV transmission from an allograft is 1 in 1,667,600.
**** The theoretical risk of developing Creutzfeldt-Jakob disease from a non-neural allograft is 1 in 3.5 million.
*** '''Tissue-processing techniques for allografts may disrupt the microstructure and affect their strength properties'''
**** '''Maximum load to failure, maximum load/graft width, and stiffness are significantly lower for the allograft freeze-dried fascia lata group compared with the autologous, solvent-dehydrated, and dermal graft groups'''
** '''Xenograft PVS'''
*** The forms of xenograft used are porcine dermis, porcine small intestinal submucosa, and bovine pericardium
*** '''Less tensile strength''' than allograft in situ and '''highest propensity to encapsulate'''
** '''Synthetic PVS'''
*** '''The most commonly used synthetic material for PVSs is polypropylene mesh.'''
*** '''Advantages:'''
**** '''Almost unlimited supply''' of artificial graft material in various sizes and shapes, consistency in quality, '''elimination of harvest site complications''', and decreased operative time.
**** More uniform, consistent, and durable compared with biologic grafts
**** Sterile, biocompatible, and noncarcinogenic
**** '''Lowest amount of degradation or disruption and the highest amount of fibroblast ingrowth and tissue ingrowth into the specimen'''
*** '''Disadvantages'''
**** '''Significant inflammatory and foreign body reactions'''
**** '''Higher rates of graft infection, urinary tract perforation, and vaginal exposure'''
***** '''No longer used to due risk of complications'''
* '''PVS Operative Procedure'''
** '''Patient Counseling'''
*** If a synthetic prosthetic or biologic graft material is being used, surgeons should thoroughly counsel their patients about the permanent nature of these products and the unique and sometimes serious complications related to their use.
*** '''Patients should also be counseled about the risk of transient and permanent voiding dysfunction after surgery, including (2):'''
***# '''Postoperative difficulty emptying the bladder'''
***# '''De novo urgency and frequency'''
** '''Graft Harvest for Autologous Pubovaginal Sling'''
*** '''Closure of the rectus fascia without tension is sometimes problematic. To prevent this difficulty, it is important to maintain a distance of ≥2 cm from the pubic symphysis'''
*** If undermining the fascial edges does not adequately mobilize the fascia, than interposition of a segment of synthetic mesh or graft may be necessary.
** Pubovaginal Sling Vaginal Approach
** Pubovaginal Sling Placement and Fixation
*** '''The bladder must be completely drained before passage of the Stamey needles to avoid inadvertent bladder injury'''
*** '''It is recommended to perform a cystoscopy after trocar passage to ensure integrity of the bladder and at the time of sling tensioning to visualize the bladder neck'''
*** '''A sling should never be tensioned before the weighted speculum is removed and the vaginal incision is closed.''' Tensioning before this may result in failure of the procedure due to too much or too little tension. The abdominal incision is closed after the sling is tensioned.
*** No suture fixation to the underlying periurethral fascia is necessary to anchor the sling  
** Pubovaginal Sling Postoperative Care
* '''Outcomes of PVS for Predominantly SUI'''
** '''No risk factors that consistently predict outcomes'''
** '''PVS are particularly helpful in treating ISD'''
** '''The reported cure rate of PVS surgery for recurrent SUI is excellent'''
** '''Autologous PVS'''
*** '''Continence rate after PVS ranges from 61-97%'''; wide range due to variation in outcome definition
*** Postoperative de novo or urgency incontinence rates range from 2-21%
**** '''Most common reason for failure/patient dissatisfaction relates to urgency symptoms and urgency incontinence at follow-up'''
** '''Allograft PVS'''
*** Limited outcome data, and the '''efficacy and durability of these slings are questionable'''
*** Previously reported failures coupled with the consistent success and rapid adoption of synthetic MUSs has led to abandonment of all types of cadaveric allograft at most centers.
** '''Xenograft PVS'''
*** Because of the morbidity of autologous fascial harvest, high failure rates of allograft materials, and high exposure and perforation rates with synthetic PVSs, xenografts are an attractive option.
*** In general, they are associated with a low rate of infection, exposure, and perforation owing to their incorporation into host tissue cure rates comparable to those of the autologous sling.
*** '''In RCTs, porcine dermis was associated with significantly inferior long-term cure rates compared with the autologous PVS'''
* '''Outcomes of Autologous PVS for Mixed Urinary Incontinence'''
** '''Overall, PVS remains an effective treatment option for MUI with cure rates similar to those of simple SUI.'''
*** '''The treatment of patients with mixed urgency and SUI is complicated and often involves a combination of anticholinergic therapy and surgery'''
*** PVS is an effective treatment option for stress-induced DO with cure rates similar to those of simple DO.
*** '''Anti-incontinence surgery may cure or aggravate urgency symptoms or lead to de novo urgency. This aspect of anti-incontinence surgery is unpredictable and a major cause of patient dissatisfaction'''
*** '''The presence of residual urgency is similar to de novo urgency with a PVS.'''
* '''Outcomes of Autologous PVS for Urethral Reconstruction'''
** '''Autologous PVSs in the setting of urethral reconstruction (urethral fistula, urethral diverticulum, destroyed urethra) has excellent results when compared with other surgeries for incontinence'''
* '''Voiding Dysfunction Secondary to Bladder Outlet Obstruction after PVS'''
** '''PVS is associated with higher success rate but increased risk of post-operative voiding dysfunction compared to the Burch colposuspension'''
*** '''<span style="color:#ff00ff">SISTEr (NEJM 2007)</span>'''
****Population: 655 women with SUI
****Randomized to autologous rectus fascia PVS vs. Burch colposuspension
****Results
*****Success rates higher for PVS
*****Voiding dysfunction (63% vs. 47%, ''P'' < .001), UTI, difficulty voiding, and postoperative urgency incontinence higher for PVS
****[https://pubmed.ncbi.nlm.nih.gov/17517855/ Albo, Michael E., et al. "Burch colposuspension versus fascial sling to reduce urinary stress incontinence." ''New England Journal of Medicine'' 356.21 (2007): 2143-2155.]
** '''The presentation of patients with obstruction by a PVS is variable and the symptoms range from complete urinary retention, impaired detrusor, detrusor overactivity contractility and urgency incontinence to the less obvious irritative symptoms.'''
*** '''Persistent/increasing urgency incontinence and urgency (8-25%) are more common presenting symptoms in bladder outlet obstruction after a PVS procedure than frank retention'''
*** '''The incidence of permanent retention is usually ≤ 5%;''' the majority of patients who require clean intermittent catheterization after PVS placement had a neurogenic bladder preoperatively
** '''There are no well-established risk factors for patients who are likely to experience voiding dysfunction after PVS surgery. Although urodynamic studies are useful in understanding the voiding dynamics of incontinent women,''' '''low detrusor pressure and Valsalva voiding preoperatively should not exclude patients from having an anti-incontinence procedure.'''
** '''Although postoperative urgency and urgency incontinence (voiding dysfunction) are strongly related to failure, there are no preoperative risk factors that consistently predict these outcomes after PVS surgery.'''
** Cystoscopy is useful to rule out bladder pathology, sling perforation, and a hypersuspended urethra.
** '''The most important criterion for a sling incision or urethrolysis remains the temporal relationship between the symptoms and the surgical procedure. Urodynamic studies are essential in these cases to diagnose and make an appropriate treatment plan.'''
** '''A key factor in assessing voiding dysfunction is the presence of prolapse that was either uncorrected at time of surgery or that occurred postoperatively.''' Prolapse of sufficient size may kink or angulate and externally compress the urethra. '''After surgery, apical, anterior, and posterior prolapse must be ruled out as a cause of the urethral obstruction.'''
** '''Surgical Management of Voiding Dysfunction after PVS'''
*** '''See 2017 AUA Female SUI Guidelines Notes'''
*** Obstruction after an autologous PVS procedure usually improves or resolves with time; '''although transient urinary retention is common, most patients return to spontaneous voiding within the first 10 days'''
*** '''In the first 6 weeks after autologous PVS surgery, loosening the sling in the operating room''' (using spinal or general anesthesia) '''can be attempted.''' This is done by first inserting a cystoscope into the bladder and then gently applying caudal pressure to the urethra. '''This procedure is not advised with synthetic slings'''
*** '''After 6 weeks or when conservative measures fail, a formal urethrolysis''' by a retropubic, transvaginal, or suprameatal approach '''or sling incision is indicated.'''
**** '''Sling incision has comparable success rates and shorter operative time and less morbidity than formal urethrolysis'''
**** Reported success rates of the surgical management of bladder outlet obstruction after a PVS procedure are 65-93%
**** Recurrent SUI after formal urethrolysis is reported as 0-19% and 34% after sling incision
*** There are no preoperative or urodynamic parameters that consistently predict success or failure of urethrolysis.
*** '''Failure of urethrolysis may be caused by persistent or recurrent obstruction, DO, impaired detrusor contractility, or learned voiding dysfunction. The most common reason for failure is likely insufficient dissection and lysis of the urethra. This supports the use of repeat urethrolysis in the face of initial failure or in cases wherein the aggressiveness of the initial dissection is unknown.'''
*** OAB symptoms are refractory in 50% of affected patients after urethrolysis and contribute to a significant portion of the reported failures. Refractory storage symptoms after urethrolysis can be challenging to treat. In addition to anticholinergics, SNM should be considered as an option for de novo or refractory urgency and urgency incontinence after urethrolysis.
* '''Complications of Pubovaginal Slings'''
** '''Mesh located in the lower urinary tract is termed a ''perforation,'' and extrusion of mesh through the skin or vagina is termed ''exposure'''''
** '''Pubovaginal Sling Perforation and Exposure'''
*** The incidence of PVS perforation and exposure is partially dependent on the composition of sling material.
**** Synthetic slings perforate 15x more often into the urethra and are exposed 14x more often in the vagina than autologous, allograft, and xenograft slings. '''The perforation or exposure of autologous PVSs is rare'''
***** Urethral perforation rate was 0.02% and the vaginal exposure rate was 0.007% in 1515 patients who received synthetic slings.
***** Urethral perforation incidence of 0.003% and a vaginal exposure incidence of 0.0001% in 1715 patients undergoing autologous and allograft sling procedures.
*** '''Urethral perforations present at ≈9 months'''
*** '''Presenting symptoms often include urinary retention, urgency, and mixed incontinence. In addition, synthetic sling perforations and exposures are also associated with vaginal discharge, vaginal pain, suprapubic pain, and recurrent UTIs.'''
*** '''Management of autologous and allograft PVS urethral perforation usually involves incision or excision of the part of the sling that has perforated and simple closure of the urethra'''
*** The incidence of recurrent SUI after synthetic PVS urethral perforation is 44-100%, and treatment often involves a second PVS
*** '''Because urinary tract perforation and vaginal exposure of synthetic PVSs are more common and associated with significant morbidity, synthetic material is no longer used for bladder neck slings'''
** '''Non-urologic Complications of PVS'''
*** '''Most commonly pulmonary, cardiovascular, neurologic, and gastrointestinal (bowel injury)'''
* '''Pubovaginal slings (PVS) are placed under mild tension at the bladder neck to reestablish the suburethral hammock and are able to improve SUI by providing a layer of tissue that compresses the urethra during times of increased intra-abdominal pressure.'''
* '''Unlike the PVS, the MUS should be placed loosely at the midportion of the urethra to prevent movement of the posterior urethral wall'''


== Midurethral Slings (MUS) ==
====== <span style="color:#ff0000">Classification (3):</span> ======
#'''<span style="color:#ff0000">Retropubic MUS (RMUS, e.g. TVT-R);</span>''' top-down or bottom-up
#'''<span style="color:#ff0000">Transobturator MUS (TMUS e.g. TVT-O);</span>''' inside-out or outside-in
#* TVT without specification refers to TVT-R
#'''<span style="color:#ff0000">Single incision sling (SIS)/adjustable sling types</span>'''
====== Mechanism of Action (2) ======
#'''<span style="color:#ff0000">Restricts movement of the posterior urethral wall above the sling</span>''' , directing its motion in an anteroinferior or anterior direction.
#*'''<span style="color:#ff0000">Patients without urethral hypermobility do not respond as well to MUS surgery'''
#** '''<span style="color:#ff0000">Lack of urethral mobility is an indication that the patient has a fixed urethra and ISD'''
#'''<span style="color:#ff0000">Narrows (compresses) the urethral lumen</span> due to inward movement of the posterior urethral wall after placement of a MUS'''


* Retropubic and transobturator MUSs are equally efficacious and generally safe
====== Anatomy ======
* '''Mechanics, Anatomy, and Materials of Midurethral Slings'''
*'''<span style="color:#ff0000">Anatomy of the Retropubic Midurethral Sling'''
** '''Mechanics of the Midurethral Sling'''
*** '''MUS works by impeding the movement of the posterior urethral wall above the sling, directing its motion in an anteroinferior or anterior direction. In addition, inward movement of the posterior urethral wall after placement of a MUS results in urethral lumen narrowing (compression).'''
*** '''Ideally placed loosely at the midurethra'''
*** '''Overall, patients without urethral hypermobility do not respond as well to MUS surgery.'''
**** '''Lack of urethral mobility is an indication that the patient has a fixed urethra and ISD.'''
*** '''A loosely placed MUS combined with a mobile urethra may allow the sling to compress the urethra during times of Valsalva and stress while remaining nonobstructive when the urethra is at rest.'''
*** Types: retropubic, transobturator, and single-incision
**** Unlike retropubic and transobturator MUSs, for single-incision MUSs it appears that restriction of urethral mobility after surgery is associated with a better outcome
* '''Anatomy of the Retropubic Midurethral Sling'''
** '''The left and right dorsal nerves of the clitoris (DNCs) run along the inferior surface of the ischiopubic rami''' and cross under the pubic bone approximately 1.4 cm from the midline
** '''The left and right dorsal nerves of the clitoris (DNCs) run along the inferior surface of the ischiopubic rami''' and cross under the pubic bone approximately 1.4 cm from the midline
** '''The obturator vessels are the closest major vascular structures to a retropubic MUS'''
** '''<span style="color:#ff0000">The obturator vessels are the closest major vascular structures to a retropubic MUS'''
* '''Anatomy of the Transobturator Midurethral Sling'''
* '''<span style="color:#ff0000">Anatomy of the Transobturator Midurethral Sling'''
** '''The transobturator technique is unique because (when done correctly) it avoids entry into the true pelvis and the levator group.'''
** '''The transobturator technique is unique because (when done correctly) it avoids entry into the true pelvis and the levator group.'''
*** '''Muscle’s traversed by this technique include the obturator internus muscle, obturator membrane, and obturator externus muscle as it goes through the obturator foramen, and the adductor muscles (adductor magnus, adductor brevis, and gracilis), lateral to the obturator foramen'''
*** '''Muscle’s traversed by this technique include the obturator internus muscle, obturator membrane, and obturator externus muscle as it goes through the obturator foramen, and the adductor muscles (adductor magnus, adductor brevis, and gracilis), lateral to the obturator foramen'''
**** '''Errant sling placement through the adductor longus tendon can result in substantial pain.'''
**** '''<span style="color:#ff0000">Errant sling placement through the adductor longus tendon can result in substantial pain.'''
** '''The obturator vessels are lateral and superior to the area of insertion of the device.'''
** '''The obturator vessels are lateral and superior to the area of insertion of the device.'''
** '''The dorsal nerve of the clitoris is separated from the trajectory of the device by ≥1-2 cm'''
** '''The dorsal nerve of the clitoris is separated from the trajectory of the device by ≥1-2 cm'''
* '''Anatomy of the Single-Incision Midurethral Slings'''
 
* '''Midurethral Sling Materials'''
====== Synthetic Sling Materials ======
** The initial MUSs were made of materials with smaller pore sizes
* The initial MUSs were made of materials with smaller pore sizes
** '''Currently, a soft, loosely woven, polypropylene monofilament mesh with a pore size > 75 μm is the most commonly used material'''
* '''Currently, a soft, loosely woven, polypropylene monofilament mesh with a pore size > 75 μm is the most commonly used material'''
* '''Midurethral Sling Operative Procedures'''
**Trend in the contemporary literature toward the use of '''macroporous polypropylene slings'''
** '''Patient Counseling'''
***'''The increased pore size of these materials allows for: excellent tissue ingrowth, promotes integration with the surrounding host tissues, and decreases encapsulation and infection'''
*** Because MUS surgery involves the implantation of a synthetic, prosthetic material, surgeons should thoroughly counsel their patients about the permanent nature of these products and the unique and sometimes serious complications related to their use. Also, similar to the PVS, patients should also be counseled about the risk of transient and permanent voiding dysfunction after surgery. This should include a '''discussion of postoperative difficulty emptying the bladder and de novo urgency and frequency.'''
 
** '''Anesthesia, Patient Positioning, and Preparation Surgical Approach for Retropubic Midurethral Slings'''
====== Contraindications ======
*** '''Cystoscopy is performed to exclude trocar penetration of the lower urinary tract.'''
*'''<span style="color:#ff0000">Should not utilize a synthetic MUS in patients undergoing concomitant (3):</span>'''
*** '''If bladder perforation is noted, the trocar is withdrawn and passed once more with an effort to avoid further perforation.'''
*#'''<span style="color:#ff0000">Urethral diverticulectomy</span>'''
*** In general, a MUS should be placed loosely at the midurethra because its function is not primarily related to compression.
*#'''<span style="color:#ff0000">Repair of urethrovaginal fistula</span>'''
** '''Surgical Approach for Transobturator out-to-in Slings'''
*# '''<span style="color:#ff0000">Urethral mesh excision</span>'''
** '''Surgical Approach for Transobturator in-to-out Slings'''
**Mesh placed in close proximity to a concurrent urethral incision can theoretically affect wound healing, potentially resulting in mesh perforation.
** '''Surgical Approach for Single-Incision Slings'''
**Instead, an anti-incontinence procedure that does not involve placement of synthetic material suburethrally, or use of a biologic material, preferably autologous fascia, should be considered.
*** '''Tension of single-incision MUSs should be tighter than the classic retropubic or transobturator MUS surgeries to achieve the same result'''
*'''<span style="color:#ff0000">Consider avoiding the use of mesh in patients undergoing SUI surgery who are at risk for poor wound healing, including</span>'''
* '''Outcomes of MUS for Predominantly SUI'''
*#'''<span style="color:#ff0000">Following radiation therapy</span>'''  
** '''Outcomes of transobturator MUS procedures in patients with predominantly SUI are similar to those of the retropubic MUS, regardless of urethral function'''
*#'''<span style="color:#ff0000">Presence of significant scarring</span>'''
** '''5-year results demonstrate durability similar to 1-year results'''
*#'''<span style="color:#ff0000">Poor tissue quality</span>'''
* '''Outcomes for Single-Incision MUS in Patients with Predominantly SUI'''
*#'''<span style="color:#ff0000">Long-term steroid use</span>'''
** There are fewer data available regarding the safety and efficacy of this new generation of slings compared with the retropubic and transobturator MUSs.
*#'''<span style="color:#ff0000">Impaired collagen associated with systemic autoimmune disorders, such as visceral Sjogren’s disease or systemic lupus erythematosus</span>'''
** '''Single-incision slings have decreasing efficacy with longer follow-up.'''
*#'''<span style="color:#ff0000">Immune suppression</span>'''
* '''Outcomes of MUS for MUI'''
====== Technique ======
* '''<span style="color:#ff0000">Should be placed loosely at the midurethra</span>''' because its function is not primarily related to compression.
**'''A loosely placed MUS combined with a mobile urethra may allow the sling to compress the urethra during times of Valsalva and stress while remaining nonobstructive when the urethra is at rest.'''
***'''For single-incision MUSs, tension should be tighter than the classic retropubic or transobturator MUS surgeries to achieve the same result'''
****Unlike retropubic and transobturator MUSs, for single-incision MUSs it appears that restriction of urethral mobility after surgery is associated with a better outcome
*'''<span style="color:#ff0000">Cystoscopy is performed to exclude trocar penetration of the lower urinary tract.'''
** '''<span style="color:#ff0000">If bladder perforation is noted, the trocar is withdrawn and passed once more with an effort to avoid further perforation'''
 
====== Outcomes ======
*'''Predominantly SUI'''
** '''<span style="color:#ff0000">Retropubic vs. Trans-obturator synthetic midurethral sling</span>'''
***'''Similar outcomes, regardless of urethral function'''
***'''Long-term comparisons are relatively lacking, however, data from increasing follow up appear to be demonstrating a <span style="color:#ff0000">lack of durability of Transobturator MUS, compared to Retropubic MUS</span>'''
** '''Single-Incision MUS in Patients with Predominantly SUI'''
*** There are fewer data available regarding the safety and efficacy of this new generation of slings compared with the retropubic and transobturator MUSs.
*** '''Single-incision slings have decreasing efficacy with longer follow-up.'''
* '''MUI'''
** '''Results with mixed incontinence are acceptable compared with other types of interventions for urinary incontinence but are less than those obtained in pure SUI.'''
** '''Results with mixed incontinence are acceptable compared with other types of interventions for urinary incontinence but are less than those obtained in pure SUI.'''
* '''Outcomes of MUS for Intrinsic Sphincteric Deficiency'''
* '''Intrinsic Sphincteric Deficiency'''
** '''The success of MUSs is lower in patients with ISD/fixed urethra (no urethral mobility) and low leak point pressures'''
** '''The success of MUSs is lower in patients with ISD/fixed urethra (no urethral mobility) and low leak point pressures'''
*** '''Fixed urethras have poor outcomes after MUS surgery regardless of leak point pressure'''
*** '''Fixed urethras have poor outcomes after MUS surgery regardless of leak point pressure'''
Line 240: Line 253:
** Low leak point pressures are not necessarily a contraindication to retropubic MUS surgery
** Low leak point pressures are not necessarily a contraindication to retropubic MUS surgery
** '''MUS is beneficial in the management of SUI in patients with ISD as long as there is preoperative urethral mobility; urethral mobility before MUS procedures has been shown to be predictive of success; the more the proximal urethra moves during a Valsalva maneuver, the better the cure rate for incontinence'''
** '''MUS is beneficial in the management of SUI in patients with ISD as long as there is preoperative urethral mobility; urethral mobility before MUS procedures has been shown to be predictive of success; the more the proximal urethra moves during a Valsalva maneuver, the better the cure rate for incontinence'''
* '''Outcomes of MUS in Patients with Pelvic Organ Prolapse'''
* '''Efficacy and safety of MUSs are not compromised in those undergoing concomitant vaginal surgery, the elderly, or the obese.'''
** '''Results suggest that the MUS can be added to prolapse surgery with minimal morbidity.'''
**'''Pelvic Organ Prolapse'''
*** The Cochrane Incontinence Group reviewed 22 randomized trials of surgical prolapse repair including 2368 women. They concluded that '''the addition of a retropubic MUS to endopelvic fascial plication, Burch colposuspension, and abdominal sacrocolpopexy may reduce the incidence of postoperative SUI''', but issues of cost and associated adverse effects were unclear
*** '''Results suggest that the MUS can be added to prolapse surgery with minimal morbidity.'''
*** Women who underwent prolapse repair at the time of the sling surgery were significantly more likely to be diagnosed with postoperative outlet obstruction (9.4% vs. 5.5%, ''P'' < .007), but less likely to undergo a repeat procedure for stress incontinence or reoperation for prolapse within 1 year after sling surgery.
**** The Cochrane Incontinence Group reviewed 22 randomized trials of surgical prolapse repair including 2368 women. They concluded that '''the addition of a retropubic MUS to endopelvic fascial plication, Burch colposuspension, and abdominal sacrocolpopexy may reduce the incidence of postoperative SUI''', but issues of cost and associated adverse effects were unclear
*** '''When MUSs are placed for urodynamic or occult SUI at time of prolapse repair, the risk of intervention because of obstruction is equivalent to the risk of intervention for SUI if no MUS was placed''' (8.5% and 8.3%, respectively)
**** Women who underwent prolapse repair at the time of the sling surgery were significantly more likely to be diagnosed with postoperative outlet obstruction (9.4% vs. 5.5%, ''P'' < .007), but less likely to undergo a repeat procedure for stress incontinence or reoperation for prolapse within 1 year after sling surgery.
** MUSs placed with either transvaginal or laparoscopic-assisted vaginal hysterectomy and anterior or posterior colporrhaphy have been shown to have success rates similar to those in published series of MUS surgery alone. Complication rates are also in accordance with other MUS series
**** '''When MUSs are placed for urodynamic or occult SUI at time of prolapse repair, the risk of intervention because of obstruction is equivalent to the risk of intervention for SUI if no MUS was placed''' (8.5% and 8.3%, respectively)
* '''Outcomes of MUS in Elderly Patients'''
*** MUSs placed with either transvaginal or laparoscopic-assisted vaginal hysterectomy and anterior or posterior colporrhaphy have been shown to have success rates similar to those in published series of MUS surgery alone. Complication rates are also in accordance with other MUS series
** '''Cure rates in older women with urethral hypermobility are comparable to those in younger women.'''
** '''Elderly Patients'''
*** '''Elderly women should not be excluded from potentially curative MUS surgery based on their age alone.'''
*** '''Cure rates in older women with urethral hypermobility are comparable to those in younger women.'''
*** Mixed urinary incontinence resolution rates are similar to those of the younger population
**** '''Elderly women should not be excluded from potentially curative MUS surgery based on their age alone.'''
*** Possibly because of preoperative factors such as MUI or even decreased urethral hypermobility, '''the rate of persistent SUI after retropubic or transobturator MUS procedures appears higher in the elderly population [different than above?]'''.
**** Mixed urinary incontinence resolution rates are similar to those of the younger population
** Complication rates vary, with some studies citing a higher rate of age-related morbidities but '''no apparent increase in intraoperative complications.'''
**** Possibly because of preoperative factors such as MUI or even decreased urethral hypermobility, '''the rate of persistent SUI after retropubic or transobturator MUS procedures appears higher in the elderly population'''.
*** '''Elderly patients experience higher rates of postoperative de novo urgency and urgency incontinence associated with any sling material, including MUS'''
*** Complication rates vary, with some studies citing a higher rate of age-related morbidities but '''no apparent increase in intraoperative complications.'''
*** Postoperative retention occurs to a similar degree as in younger patients
**** '''Elderly patients experience higher rates of postoperative de novo urgency and urgency incontinence associated with any sling material, including MUS'''
* '''Outcomes of MUS in Obese Patients'''
**** Postoperative retention occurs to a similar degree as in younger patients
** '''Whether obesity affects surgical outcome with MUSs is controversial.'''
** '''Obese Patients'''
** Overall, the rate of complications appears to be similar in obese versus non-obese patients undergoing MUS surgery.
*** '''Controversial whether obesity affects surgical outcome with MUSs'''
*** Higher rate of bladder trocar injury in non-obese patients
*** Overall, the rate of complications appears to be similar in obese versus non-obese patients undergoing MUS surgery.
* '''Efficacy and safety of MUSs are not compromised in the elderly, the obese, or those undergoing concomitant vaginal surgery.'''
**** Higher rate of bladder trocar injury in non-obese patients
* '''Outcomes of MUS for Recurrent Stress Urinary Incontinence'''
* '''Recurrent Stress Urinary Incontinence'''
** '''As salvage procedures, MUS have overall efficacy similar to their use in primary implantation procedures.'''
** '''As salvage procedures, MUS have overall efficacy similar to their use in primary implantation procedures.'''
** '''The procedure can be performed in the same way as it is performed for primary SUI.'''
** '''The procedure can be performed in the same way as it is performed for primary SUI.'''
Line 265: Line 278:
** As is the case with primary surgery, the failure rate is higher in females with immobile urethras.
** As is the case with primary surgery, the failure rate is higher in females with immobile urethras.
** '''No significant difference in subjective cure rates in patients after retropubic vs. transobturator MUS surgery for recurrent SUI.'''
** '''No significant difference in subjective cure rates in patients after retropubic vs. transobturator MUS surgery for recurrent SUI.'''
* '''Complications of Midurethral Slings'''
 
** Overall, the rate of complications associated with the MUS procedure is relatively low.
====== <span style="color:#ff0000">Adverse events ======
** '''The rates of:'''
* '''Patient counselling'''
**# '''Bladder trocar injury: 2.7-3.8%; higher with retropubic'''
**'''Permanent nature of these products since''' MUS surgery involves the implantation of a synthetic, prosthetic material
**# '''Voiding dysfunction: 7.6%'''
**'''Unique and sometimes serious complications related to their use'''
**# '''Wound healing problems: 1%'''
**'''Risk of''' '''transient and permanent voiding dysfunction after surgery, including''' '''postoperative difficulty emptying the bladder and de novo urgency and frequency'''
**# '''Vaginal mesh exposure: 0.5-8.1%'''
*'''<span style="color:#ff0000">Overall, relatively low rates of complications (10)'''
**# '''Mesh perforation of urethra''' (transobturator and retropubic): '''0-0.6%'''
*# '''<span style="color:#ff0000">Bladder trocar injury: 2.7-3.8%; higher with retropubic'''
**# '''Mesh perforation of bladder: 0.5-0.6%'''
*# '''<span style="color:#ff0000">Voiding dysfunction: 7.6%'''
**# '''Sexual dysfunction'''
*# '''<span style="color:#ff0000">Wound healing problems: 1%'''
**# '''Groin pain; higher with transobturator'''; conservative therapy with NSAIDs should resolve the majority of symptoms. If pain persists after 6-8 weeks, consider referral to a pain clinic for trigger point injections and physical therapy.
*# '''<span style="color:#ff0000">Vaginal mesh exposure</span>: 0.5-8.1%'''
**# '''Bleeding, vascular injury; higher with retropubic'''
*# '''<span style="color:#ff0000">Mesh perforation of urethra</span>''' (transobturator and retropubic): '''0-0.6%'''
**# '''Infection, bowel perforation, and death'''
*# '''<span style="color:#ff0000">Mesh perforation of bladder</span>: 0.5-0.6%'''
** The exact cause of these complications is debatable, but they likely arise from a combination of patient and technical factors.
*# '''<span style="color:#ff0000">Sexual dysfunction'''
*** Factors include patient body habitus, subclinical infection, poor tissue ingrowth into the sling, disturbed wound healing, rolling or twisting of the sling, excessive friction between host tissue and the sling, sling material properties, and iatrogenic injury and surgeon technical error.
*# '''<span style="color:#ff0000">Groin pain; higher with transobturator</span>'''; conservative therapy with NSAIDs should resolve the majority of symptoms. If pain persists after 6-8 weeks, consider referral to a pain clinic for trigger point injections and physical therapy.
*** Biomechanical properties of the sling material have also been shown to play a major role in the incidence of complications related to mesh exposure. Although various materials have been historically used for sling implants, there has been a trend in the contemporary literature toward the use of '''macroporous polypropylene slings. The increased pore size of these materials allows for: excellent tissue ingrowth, promotes integration with the surrounding host tissues, and decreases encapsulation and infection'''
*# '''<span style="color:#ff0000">Bleeding, vascular injury; higher with retropubic'''
** '''MUS Bladder Trocar Injury'''
*# '''Infection, bowel perforation, and death'''
*** '''Rate of trocar injury is higher with retropubic vs. trans-obturator MUS'''
* '''<span style="color:#ff0000">Bladder Trocar Injury'''
**** '''The rate of bladder or urethral trocar injury:'''
** '''<span style="color:#ff0000">Rate of trocar injury is higher with retropubic vs. trans-obturator MUS'''
***** '''Retropubic MUS surgery: 2.7-23.8%'''
*** '''The rate of bladder or urethral trocar injury:'''
***** '''Trans-obturator MUS surgery: 0-1.3%'''
**** '''Retropubic MUS surgery: 2.7-23.8%'''
*** '''Trocar injury is generally thought of as a benign condition. If occurs intra-operatively, remove trocar and pass trocar again more laterally''' and leave foley catheter in 3-7 days.
**** '''Trans-obturator MUS surgery: 0-1.3%'''
** '''MUS Mesh Exposure'''
** '''<span style="color:#ff0000">Management'''
*** '''Most cases manifest within a few weeks to a few months after the MUS procedure'''
***'''<span style="color:#ff0000">If occurs intra-operatively, remove trocar and pass trocar again more laterally and leave foley catheter in 3-7 days.'''
*** '''Symptoms of vaginal exposure include:'''
****'''Trocar injury is generally thought of as a benign condition.'''
***# '''Vaginal discharge''' (with variable constituents and different amounts of blood and inflammatory components)
* '''<span style="color:#ff0000">Vaginal Mesh Exposure'''
***# '''Palpable rough surface in the vagina'''
** '''Most cases manifest within a few weeks to a few months after the MUS procedure'''
***# '''Sexual discomfort''' (including partner related)
** '''Risk factors (5):'''
***# '''Pelvic pain'''
**#'''Diabetes'''
***# '''Inguinal discomfort'''
**#'''History of smoking'''
***# '''LUTS (urgency, frequency, persistent incontinence, hematuria)'''
**#'''Older age'''
**#'''>2 cm vaginal incision length'''
**#'''Previous vaginal surgery'''
**'''<span style="color:#ff0000">Diagnosis and Evaluation'''
***'''<span style="color:#ff0000">History and Physical Exam'''
****'''<span style="color:#ff0000">Signs and Symptoms (6):'''
****# '''<span style="color:#ff0000">Vaginal discharge</span>''' (with variable constituents and different amounts of blood and inflammatory components)
****# '''<span style="color:#ff0000">Palpable rough surface in the vagina'''
****# '''<span style="color:#ff0000">Sexual discomfort</span>''' (including partner related)
****# '''<span style="color:#ff0000">Pelvic pain'''
****# '''<span style="color:#ff0000">Inguinal discomfort'''
****# '''<span style="color:#ff0000">LUTS (urgency, frequency, persistent incontinence, hematuria)'''
** '''<span style="color:#ff0000">Management'''
*** The management of this complication is not standardized, composition of mesh is particularly important in the event of mesh exposure
*** The management of this complication is not standardized, composition of mesh is particularly important in the event of mesh exposure
*** '''Management'''
***'''<span style="color:#ff0000">Options (2)'''
**** '''Conservative management is an option in well-selected patients who are relatively asymptomatic and have small-caliber exposures (<1 cm)'''
****'''<span style="color:#ff0000">Conservative management with conjugated estrogen and possibly antibiotic creams'''
**** '''The next step involves addition of conjugated estrogen and possibly antibiotic creams'''
*****'''<span style="color:#ff0000">Option in well-selected patients who are relatively asymptomatic and have small-caliber exposures (<1 cm)'''
**** '''Excision should be reserved for failure of conservative therapy or when local symptoms mitigate against observational management (e.g., bothersome dyspareunia).''' '''Limited excision and trimming with vaginal closure can be attempted.''' '''Even with partial excision of the mesh, continence is maintained in the majority of patients.'''
**** '''<span style="color:#ff0000">Excision should be reserved for failure of conservative therapy or when local symptoms mitigate against observational management (e.g., bothersome dyspareunia).'''  
**** '''If these options fail, excision of most of the mesh from a transvaginal approach should be pursued in most cases.'''
*****'''Limited excision and trimming with vaginal closure can be attempted.'''  
***** '''Operative management typically involves excision of the exposed mesh''', thorough irrigation with antibiotic solution, and closure of vaginal flaps. Good results have also been observed in selected patients with vaginal advancement flaps and suture approximation of the debrided vaginal mucosa over the exposed mesh.
******'''Even with partial excision of the mesh, continence is maintained in the majority of patients.'''
* '''MUS Mesh Perforation of the Urethra'''
***** '''If these options fail, excision of most of the mesh from a transvaginal approach should be pursued in most cases.'''
** '''Presenting symptoms are variable, voiding dysfunction is predominant'''
****** '''Operative management typically involves excision of the exposed mesh''', thorough irrigation with antibiotic solution, and closure of vaginal flaps. Good results have also been observed in selected patients with vaginal advancement flaps and suture approximation of the debrided vaginal mucosa over the exposed mesh.
** '''Management'''
* '''<span style="color:#ff0000">Mesh Perforation of the Urethra'''
** '''<span style="color:#ff0000">Diagnosis and Evaluation'''
***'''History and Physical Exam'''
****'''History'''
*****'''Presenting symptoms are variable'''
*****'''Voiding dysfunction is predominant'''
** '''<span style="color:#ff0000">Management'''
*** Management of this complication is extremely challenging
*** Management of this complication is extremely challenging
*** '''Observation should never be considered when there is urethral perforation'''
*** '''<span style="color:#ff0000">Observation should never be considered when there is urethral perforation'''
*** '''Although we prefer transvaginal surgical excision as a first-line treatment, endoscopic management of small areas of mesh perforation appears to be a reasonable initial option'''
*** '''Options (2)'''
*** '''For slings that perforate into the urethra an inverted-U incision is best''' because this allows for exposure of the proximal urethra, bladder neck, and endopelvic fascia as well as providing a vaginal epithelial flap that avoids overlapping suture lines
***#'''Endoscopic management of small areas of mesh perforation'''
*** An autologous fascial sling or a Martius labial fat pad graft can be used, at the discretion of the surgeon.
***#'''Transvaginal surgical excision'''
*** '''An autologous fascial sling can be placed at the time of surgery to augment the repair or in a delayed fashion to treat recurrent SUI.'''
***#* '''For slings that perforate into the urethra an inverted-U incision is best''' because this allows for exposure of the proximal urethra, bladder neck, and endopelvic fascia as well as providing a vaginal epithelial flap that avoids overlapping suture lines
* '''MUS Mesh Perforation of the Bladder'''
***#* An autologous fascial sling or a Martius labial fat pad graft can be used for repair, at the discretion of the surgeon.
** The majority of intravesical mesh perforations are most likely the result of an unrecognized cystotomy or placement of the mesh within the urinary bladder at the time of surgery.
***#** '''An autologous fascial sling can be placed at the time of surgery to augment the repair or in a delayed fashion to treat recurrent SUI.'''
** Typical symptoms are lower abdominal pain, intermittent gross hematuria, recurrent UTI, urgency, frequency, dysuria, and urinary incontinence.
* '''<span style="color:#ff0000">Mesh Perforation of the Bladder'''
** '''Management of MUS Mesh Perforation of the Bladder'''
** '''Pathogenesis'''
*** '''Observation should never be considered when there is intravesical perforation'''
***'''Most likely the result of an unrecognized cystotomy or placement of the mesh within the urinary bladder at the time of surgery'''.
*** '''For small areas of mesh perforation, endoscopic excision with scissors or ablation with the holmium laser as an appropriate initial step, although this is contingent on absolute excision of all exposed material'''
** '''<span style="color:#ff0000">Diagnosis and Evaluation'''
*** '''After endoscopic excision fails or as an initial treatment, mesh perforated into the bladder can be removed from a transvaginal or retropubic approach.'''
***'''History and Physical Exam'''
*** Reconstruction should involve nonoverlapping suture lines and interposition of tissue such as a labial fat pad or greater omentum.
****'''Signs and Symptoms'''
*** An autologous fascial sling can be placed at the time of surgery to augment the repair or in a delayed fashion to treat recurrent SUI.
****#'''Lower abdominal pain'''
****#'''Intermittent gross hematuria'''
****#'''Recurrent UTI'''
****#'''Urgency'''
****#'''Frequency'''
****#'''Dysuria'''
****#'''Urinary incontinence'''
** '''<span style="color:#ff0000">Management'''
*** '''<span style="color:#ff0000">Observation should never be considered when there is intravesical perforation'''
***'''Options (2)'''
***#'''If small area of mesh perforation: endoscopic management (excision with scissors or ablation with the holmium laser) is an appropriate initial step if complete excision of all exposed material can be achieved'''
***# '''If endoscopic excision fails or as initial treatment for large areas of mesh perforation, transvaginal or retropubic excision'''
***## Reconstruction should involve nonoverlapping suture lines and interposition of tissue such as a labial fat pad or greater omentum.
***## An autologous fascial sling can be placed at the time of surgery to augment the repair or in a delayed fashion to treat recurrent SUI.
* '''Pain and Infection after MUS Surgery'''
* '''Pain and Infection after MUS Surgery'''
** Groin and suprapubic pain are potential problems after MUS placement. '''Thigh and groin pain appear to be more commonly associated with the transobturator approach'''
** '''Groin and suprapubic pain are potential problems after MUS placement.'''
*** Most groin pain resolves after the second postoperative day.
***'''Thigh and groin pain appear to be more commonly associated with the transobturator approach'''
**** Pain persists longer after the transobturator midurethral slings.
** '''Severe infection is a rare complication after MUS surgery'''
** Severe infection is a rare complication after MUS surgery, and the diagnosis of this complication is variable and can take as long as several years
***Diagnosis of this complication is variable and can take as long as several years
*** '''Obesity, diabetes, and hypertension are associated with fasciitis after pelvic surgery.'''
*** '''Obesity, diabetes, and hypertension are associated with fasciitis after pelvic surgery.'''
** Management of MUS Severe Infection or Pain.
** '''Management of MUS Severe Infection or Pain'''
*** In most cases, postoperative groin or leg pain after MUS surgery can be managed with NSAIDs, rest, and physical therapy. Most groin pain resolves after postoperative day 2
*** '''In most cases, postoperative groin or leg pain after MUS surgery can be managed with NSAIDs, rest, and physical therapy.'''
****'''Most groin pain resolves after postoperative day 2'''
*****Pain persists longer after the transobturator midurethral slings
*** In instances of chronic mesh pain and severe infection when nonoperative therapy has failed, it may be necessary to attempt a complete mesh excision from both sides of the bone.
*** In instances of chronic mesh pain and severe infection when nonoperative therapy has failed, it may be necessary to attempt a complete mesh excision from both sides of the bone.
*** For the complete excision of transobturator mesh we typically consult an orthopedic surgeon to aid with lateral dissection of the sling.
*** For the complete excision of transobturator mesh we typically consult an orthopedic surgeon to aid with lateral dissection of the sling.
* '''Voiding Dysfunction after MUS Surgery'''
* '''<span style="color:#ff0000">Voiding Dysfunction after MUS Surgery'''
** '''Voiding dysfunction after MUS is substantially less than with PVS (bladder neck) slings but still occurs'''
** '''Epidemiology'''
** Usually transient, long-term urinary retention and obstructive voiding dysfunction are rare after the MUS procedure
***De novo urgency occurs with postoperative voiding dysfunction in as many as 12% of patients
** '''Typically the result of obstruction from the sling as a consequence of the sling being placed too tightly or in the wrong location (too proximally) or associated with pelvic organ prolapse (unrecognized preoperatively or de novo); however, some patients may have voiding dysfunction without evidence of obstruction'''
***'''Voiding dysfunction after MUS is substantially less than with PVS (bladder neck) slings'''
** '''In general, the rates of de novo urgency and perioperative urinary retention are similar among the different types of MUSs. The most common symptoms of obstruction are an inability to void (urinary retention), incomplete emptying, and de novo urgency and frequency.''' Over several weeks to a month, the irritative voiding symptoms (urgency, frequency, and pain) become more prevalent as the bladder attempts to adjust to the obstruction.
**'''Usually transient'''
*** De novo urgency occurs with postoperative voiding dysfunction in as many as 12% of patients
***Long-term urinary retention and obstructive voiding dysfunction are rare after the MUS procedure
*** '''However, a recent RCT found significantly higher rate of voiding dysfunction necessitating surgery (or permanent catheter) after a retropubic MUS compared with a transobturator sling procedure (2.7% vs. 0%).'''
**'''Pathogenesis'''
*** In addition, the rate of urinary retention (catheter for longer than 6 weeks) was also higher in the retropubic MUS group (3.7% vs. 0.7%).
***'''Typically the result of obstruction'''
** There does not appear to be a consensus in the literature regarding preoperative factors that contribute to voiding dysfunction after MUS surgery.
****'''From the sling being placed too tightly or in the wrong location (too proximally)'''
** The optimal evaluation for patients with postoperative voiding dysfunction is poorly defined in the literature. The decision to perform urethrolysis is usually based on a clear temporal relationship between onset of symptoms and the surgical procedure.
****'''Associated with pelvic organ prolapse (unrecognized preoperatively or de novo)'''
** Cystoscopy is useful to rule out bladder pathology, urethral mesh perforation, and a hypersuspended bladder neck.
*****Some patients may have voiding dysfunction without evidence of obstruction
** '''Management of Voiding Dysfunction after Midurethral Sling Surgery'''
** '''Risk factors'''
*** '''Urinary obstruction after MUS surgery is usually transient and can be managed with short-term intermittent catheterization'''
***'''Type of MUS'''
*** '''For patients with persistently elevated residual urine and bothersome symptoms refractory to conservative management, transvaginal sling release procedures consistently provide resolution of symptoms with maintenance of continence in the majority of patients'''
****'''In general, similar rates of de novo urgency and perioperative urinary retention among the different types of MUSs'''
**** '''Urethral dilation is of limited usefulness and, if used too aggressively, may be detrimental.'''
****'''However, an RCT found significantly higher rate of voiding dysfunction necessitating surgery (or permanent catheter) after a retropubic MUS compared with a transobturator sling procedure (3% vs. 0%)[https://www.ncbi.nlm.nih.gov/pmc/articles/PMC4367868/ §]'''
**** '''Cutting the MUS in the midline through a single vertical vaginal incision using minimal dissection is the preferred method to manage persistent voiding dysfunction that results from an obstructive sling within the first 3 months after surgery; the entire sling does not need to be excised'''
**** In addition, the rate of urinary retention (catheter for longer than 6 weeks) was also higher in the retropubic MUS group (3.7% vs. 0.7%).
***** '''The exact timing of sling incision is variable; however, most recommend waiting at least 2 weeks; the sling should be incised within 4 weeks of surgery'''
** '''<span style="color:#ff0000">Diagnosis and Evaluation'''
***** '''The majority of patients maintain continence with single incision'''
***'''<span style="color:#ff0000">History and Physical Exam'''
**** '''After 3 months, the sling may be fixed along its entire course, and midline sling incision may not achieve enough sling relaxation to resolve voiding dysfunction. In these cases, we perform a more formal sling excision and urethrolysis'''
****'''<span style="color:#ff0000">History'''
* '''Sexual Dysfunction after MUS'''
*****'''<span style="color:#ff0000">Most common symptoms of obstruction are'''
******'''<span style="color:#ff0000">Inability to void (urinary retention)'''
******'''<span style="color:#ff0000">Incomplete emptying'''
******'''<span style="color:#ff0000">De novo urgency and frequency'''
*****Over several weeks to a month, the storage symptoms (urgency and frequency) and pain become more prevalent as the bladder attempts to adjust to the obstruction.
****'''<span style="color:#ff0000">Physical Exam'''
*****'''<span style="color:#ff0000">Pelvic organ prolapse'''
******'''A key factor in assessing voiding dysfunction is the presence of prolapse that was either uncorrected at time of surgery or that occurred postoperatively.''' Prolapse of sufficient size may kink or angulate and externally compress the urethra. '''After surgery, apical, anterior, and posterior prolapse must be ruled out as a cause of the urethral obstruction'''
***'''<span style="color:#ff0000">Cystoscopy'''
****'''<span style="color:#ff0000">Useful to rule out bladder pathology, urethral mesh perforation, and a hypersuspended bladder neck'''
***The optimal evaluation for patients with postoperative voiding dysfunction is poorly defined in the literature. The decision to perform urethrolysis is usually based on a clear temporal relationship between onset of symptoms and the surgical procedure.
** '''<span style="color:#ff0000">Management'''  
*** '''<span style="color:#ff0000">Urinary obstruction after MUS surgery is usually transient and can be managed with short-term intermittent catheterization'''
*** '''<span style="color:#ff0000">If persistently elevated residual urine, bothersome symptoms refractory to conservative management, and within first 3 months of surgery, then perform transvaginal sling release'''
**** '''Cutting the MUS in the midline through a single vertical vaginal incision using minimal dissection is the preferred method to manage persistent voiding dysfunction that results from an obstructive sling within the first 3 months after surgery'''
*****'''Transvaginal sling release consistently provide resolution of symptoms with maintenance of continence in the majority of patients'''
******The entire sling does not need to be excised; majority of patients maintain continence with single incision
**** '''Timing'''
*****'''The exact timing of sling incision is variable; however, most recommend waiting at least 2 weeks; the sling should be incised within 4 weeks of surgery'''
*** '''<span style="color:#ff0000">If after 3 months, perform a more formal sling excision and urethrolysis'''
****'''Sling may be fixed along its entire course, and midline sling incision may not achieve enough sling relaxation to resolve voiding dysfunction'''
***'''<span style="color:#ff0000">Urethral dilation is of limited usefulness and, if used too aggressively, may be detrimental.'''
* '''<span style="color:#ff0000">Sexual Dysfunction after MUS'''
** '''Contradictory evidence in the literature that MUS surgery improves and worsens sexual function'''
** '''Contradictory evidence in the literature that MUS surgery improves and worsens sexual function'''
*** Some attribute improved sexual function after MUS surgery to a significant decrease in coital incontinence
*** Some attribute improved sexual function after MUS surgery to a significant decrease in coital incontinence
Line 356: Line 424:
* '''Other Complications after MUS Surgery'''
* '''Other Complications after MUS Surgery'''
** Infection, bleeding, vascular injury, bowel perforation, and death.
** Infection, bleeding, vascular injury, bowel perforation, and death.
*** '''Risk of major vascular injury higher with retropubic vs. transobturator'''
*** '''<span style="color:#ff0000">Risk of major vascular injury higher with retropubic vs. transobturator'''
*** '''The majority of postoperative hematomas resolve without intervention'''
*** '''The majority of postoperative hematomas resolve without intervention'''
** '''UTI is the most common''' '''and easily treatable complication of MUS surgery.'''
** '''<span style="color:#ff0000">UTI is the most common and easily treatable complication of MUS surgery.'''
* '''Regulatory and Legal Issues Related to Sling Mesh Complications'''
 
** See CUA/AUA Mesh Position Statements
====== Regulatory and Legal Issues Related to Sling Mesh Complications ======
** The success of MUSs for incontinence led to the development of mesh products for pelvic organ prolapse repair. However, while the use of mesh during pelvic organ prolapse repairs has increased, so too have complications related to its use.
* '''See [[AUA & CUA Mesh Position Statement (2019)|AUA/CUA Mesh Position Statements]]'''
** In October 2008, the FDA released a public health notification (PHN) alerting the public about “rare” complications and problems related to transvaginal mesh products used for pelvic organ prolapse. '''In 2011, the FDA''' modified this alert by removing the term “rare” and '''stating that surgical mesh for pelvic organ prolapse repair does not conclusively improve outcomes over traditional nonmesh or native tissue repairs and is associated with unique potentially serious adverse outcomes'''
* The success of MUSs for incontinence led to the development of mesh products for pelvic organ prolapse repair. However, while the use of mesh during pelvic organ prolapse repairs has increased, so too have complications related to its use.
** '''FDA advisory panel deemed existing MUS products “safe and effective”'''
* Synthetic mesh for pelvic organ prolapse
**In October 2008, the FDA released a public health notification (PHN) alerting the public about “rare” complications and problems related to transvaginal mesh products used for pelvic organ prolapse.  
**'''In 2011, the FDA''' modified this alert by removing the term “rare” and '''stating that surgical mesh for pelvic organ prolapse repair does not conclusively improve outcomes over traditional nonmesh or native tissue repairs and is associated with unique potentially serious adverse outcomes'''
* '''Synthetic mesh for midurethral slings'''
**'''FDA advisory panel deemed existing MUS products “safe and effective”'''
** Even though the FDA has determined that existing MUS products are safe and effective, this unfortunately does not prevent MUSs from being caught up in the fervor of litigation related to products used for pelvic organ prolapse repair.
** Even though the FDA has determined that existing MUS products are safe and effective, this unfortunately does not prevent MUSs from being caught up in the fervor of litigation related to products used for pelvic organ prolapse repair.
** '''Single-incision sling manufacturers are required to perform 522 postmarket surveillance studies'''
** '''Single-incision sling manufacturers are required to perform 522 postmarket surveillance studies'''
===== Pubovaginal Slings (PVS) =====
* '''Indicated for treatment of incontinence associated with'''
**'''Deficiency in a portion of the midurethral complex'''
**'''Hypermobility'''
**'''ISD'''
**'''MUI'''
**'''Concomitant cystoceles'''
**'''Urethral diverticula'''
**'''Neurologic conditions'''
====== Anatomy and Mechanics of a PVS ======
* '''<span style="color:#ff0000">Positioned at the bladder neck (in contrast to MUS which is placed in the midurethra)</span> to provide urethral compression without obstruction during times of increased intraabdominal pressure.'''
**Pubovaginal slings are placed under mild tension at the bladder neck to reestablish the suburethral hammock and are able to improve SUI by providing a layer of tissue that compresses the urethra during times of increased intra-abdominal pressure.
** Unlike the PVS, the MUS should be placed loosely at the midportion of the urethra to prevent movement of the posterior urethral wall
* The current concept of PVS comes from using a shorter free graft of rectus fascia whose tension could be adjusted
====== Pubovaginal Sling Materials ======
* '''Options (4):'''
*#'''Autologous'''
*#'''Allograft'''
*#'''Xenograft'''
*#'''Synthetic'''
*#* Although there is complete biocompatibility of the autologous sling and negligible urethral perforation, biologic graft and synthetic prosthetic materials have been increasingly used to decrease operative time, morbidity, pain, and hospital stay
* '''<span style="color:#ff0000">Autologous fascia PVS'''
** '''<span style="color:#ff0000">Gold standard for management of ALL forms of SUI'''
** '''Advantages compared to alternative sling materials (2):'''
**# '''Minimal tissue inflammation'''
**# '''Negligible risk of urethral erosion'''
** '''Disadvantages compared to alternative sling materials (4):'''
*** '''Increased operative time, hospital stay'''
***'''Postoperative pain'''
***'''Risk of suprapubic wound seroma'''
***'''Risk of incisional hernia'''
** '''<span style="color:#ff0000">Most commonly used autologous materials (2):'''
**# '''<span style="color:#ff0000">Rectus abdominis fascia harvested from the abdominal wall'''
**#* '''<span style="color:#ff0000">Most commonly used'''
**# '''<span style="color:#ff0000">Fascia lata harvested from the lateral thigh'''
**#* '''Fascia lata is the preferred autologous material for PVSs in patients with a history of prior ventral hernia repair;''' unlike rectus facia''',''' the recovery time is less and there is no risk of future abdominal hernia formation.
**#* Disadvantages of fascia lata compared to rectus abdominis:
**#*# Requires repositioning of the patient
**#*# Increased operative time
**#*# Operating in an area unfamiliar to most pelvic surgeons
**# '''Rectus abdominis fascia vs. fascia lata PVS: similar improvement of SUI'''
* '''Allograft PVS'''
** Were introduced in an effort to reduce overall morbidity, operative time, and pain related to graft procurement
** Currently derived from either cadaveric fascia lata or acellular human dermis
** Allografts from cadavers raise the concern of potentially transmitting illnesses such as HIV, hepatitis, and Creutzfeldt-Jakob prion disease
*** The estimated risk of HIV transmission from an allograft is 1 in 1,667,600.
*** The theoretical risk of developing Creutzfeldt-Jakob disease from a non-neural allograft is 1 in 3.5 million.
** '''Tissue-processing techniques for allografts may disrupt the microstructure and affect their strength properties'''
*** '''Maximum load to failure, maximum load/graft width, and stiffness are significantly lower for the allograft freeze-dried fascia lata group compared with the autologous, solvent-dehydrated, and dermal graft groups'''
* '''Xenograft PVS'''
** The forms of xenograft used are porcine dermis, porcine small intestinal submucosa, and bovine pericardium
** '''Less tensile strength''' than allograft in situ and '''highest propensity to encapsulate'''
* '''Synthetic PVS'''
** '''The most commonly used synthetic material for PVSs is polypropylene mesh.'''
** '''Advantages:'''
*** '''Almost unlimited supply''' of artificial graft material in various sizes and shapes, consistency in quality, '''elimination of harvest site complications''', and decreased operative time.
*** More uniform, consistent, and durable compared with biologic grafts
*** Sterile, biocompatible, and noncarcinogenic
*** '''Lowest amount of degradation or disruption and the highest amount of fibroblast ingrowth and tissue ingrowth into the specimen'''
** '''Disadvantages'''
*** '''Significant inflammatory and foreign body reactions'''
*** '''Higher rates of graft infection, urinary tract perforation, and vaginal exposure'''
**** '''No longer used to due risk of complications'''
'''Patient counselling'''
*If a synthetic prosthetic or biologic graft material is being used, surgeons should thoroughly counsel their patients about the permanent nature of these products and the unique and sometimes serious complications related to their use.
*'''Risk of''' '''transient and permanent voiding dysfunction after surgery, including''' '''postoperative difficulty emptying the bladder and de novo urgency and frequency'''
====== Technique ======
* '''Graft Harvest for Autologous Pubovaginal Sling'''
** '''Closure of the rectus fascia without tension is sometimes problematic. To prevent this difficulty, it is important to maintain a distance of ≥2 cm from the pubic symphysis'''
** If undermining the fascial edges does not adequately mobilize the fascia, than interposition of a segment of synthetic mesh or graft may be necessary.
* '''Pubovaginal Sling Placement and Fixation'''
** '''The bladder must be completely drained before passage of the Stamey needles to avoid inadvertent bladder injury'''
** '''Perform a cystoscopy after trocar passage to ensure integrity of the bladder and at the time of sling tensioning to visualize the bladder neck'''
** '''A sling should never be tensioned before the weighted speculum is removed and the vaginal incision is closed.'''
***Tensioning before this may result in failure of the procedure due to too much or too little tension. The abdominal incision is closed after the sling is tensioned.
** No suture fixation to the underlying periurethral fascia is necessary to anchor the sling  
====== Outcomes ======
* '''Predominantly SUI'''
**'''No risk factors that consistently predict outcomes'''
** '''<span style="color:#ff0000">PVS are particularly helpful in treating ISD (in contrast to midurethral sling)'''
** '''The reported cure rate of PVS surgery for recurrent SUI is excellent'''
** '''Autologous PVS'''
*** '''Continence rate after PVS ranges from 61-97%'''; wide range due to variation in outcome definition
*** Postoperative de novo or urgency incontinence rates range from 2-21%
**** '''Most common reason for failure/patient dissatisfaction relates to urgency symptoms and urgency incontinence at follow-up'''
** '''Allograft PVS'''
*** Limited outcome data, and the '''efficacy and durability of these slings are questionable'''
*** Previously reported failures coupled with the consistent success and rapid adoption of synthetic MUSs has led to abandonment of all types of cadaveric allograft at most centers.
** '''Xenograft PVS'''
*** Because of the morbidity of autologous fascial harvest, high failure rates of allograft materials, and high exposure and perforation rates with synthetic PVSs, xenografts are an attractive option.
*** In general, they are associated with a low rate of infection, exposure, and perforation owing to their incorporation into host tissue cure rates comparable to those of the autologous sling.
*** '''In RCTs, porcine dermis was associated with significantly inferior long-term cure rates compared with the autologous PVS'''
* '''Autologous PVS for Mixed Urinary Incontinence'''
** '''Overall, PVS remains an effective treatment option for MUI with cure rates similar to those of simple SUI.'''
*** '''The treatment of patients with mixed urgency and SUI is complicated and often involves a combination of anticholinergic therapy and surgery'''
*** PVS is an effective treatment option for stress-induced DO with cure rates similar to those of simple DO.
*** '''Anti-incontinence surgery may cure or aggravate urgency symptoms or lead to de novo urgency. This aspect of anti-incontinence surgery is unpredictable and a major cause of patient dissatisfaction'''
*** '''The presence of residual urgency is similar to de novo urgency with a PVS.'''
* '''Autologous PVS for Urethral Reconstruction'''
** '''<span style="color:#ff0000">Autologous PVSs in the setting of urethral reconstruction (urethral fistula, urethral diverticulum, destroyed urethra) has excellent results when compared with other surgeries for incontinence'''
====== Adverse Events ======
* '''Pubovaginal Sling Perforation and Exposure'''
** '''Incidence is partially dependent on the composition of sling material'''
*** Synthetic slings perforate 15x more often into the urethra and are exposed 14x more often in the vagina than autologous, allograft, and xenograft slings.
***'''<span style="color:#ff0000">Perforation or exposure of autologous pubovaginal slings is rare'''
**** Urethral perforation rate was 0.02% and the vaginal exposure rate was 0.007% in 1515 patients who received synthetic slings.
**** Urethral perforation incidence of 0.003% and a vaginal exposure incidence of 0.0001% in 1715 patients undergoing autologous and allograft sling procedures.
***'''Because urinary tract perforation and vaginal exposure of synthetic PVSs are more common and associated with significant morbidity, synthetic material is no longer used for bladder neck slings'''
** '''<span style="color:#ff0000">Diagnosis and Evaluation'''
***'''Urethral perforations present at ≈9 months'''
*** '''<span style="color:#ff0000">History and Physical Exam'''
****'''<span style="color:#ff0000">Presenting symptoms often include urinary retention, urgency, and mixed incontinence.'''
****'''<span style="color:#ff0000">In addition, synthetic sling perforations and exposures are also associated with vaginal discharge, vaginal pain, suprapubic pain, and recurrent UTIs.'''
** '''<span style="color:#ff0000">Management'''
***'''<span style="color:#ff0000">Management of autologous and allograft PVS urethral perforation usually involves incision or excision of the part of the sling that has perforated and simple closure of the urethra'''
*** The incidence of recurrent SUI after synthetic PVS urethral perforation is 44-100%, and treatment often involves a second PVS
* '''Voiding Dysfunction Secondary to Bladder Outlet Obstruction after PVS'''
** '''PVS is associated with higher success rate but increased risk of post-operative voiding dysfunction compared to the Burch colposuspension'''
*** '''<span style="color:#ff00ff">SISTEr (NEJM 2007)</span>'''
****Population: 655 women with SUI
****Randomized to autologous rectus fascia PVS vs. Burch colposuspension
****Results
*****Success rates higher for PVS
*****Voiding dysfunction (63% vs. 47%, ''P'' < .001), UTI, difficulty voiding, and postoperative urgency incontinence higher for PVS
****[https://pubmed.ncbi.nlm.nih.gov/17517855/ Albo, Michael E., et al. "Burch colposuspension versus fascial sling to reduce urinary stress incontinence." ''New England Journal of Medicine'' 356.21 (2007): 2143-2155.]
** '''Risk Factors'''
***'''No well-established risk factors for patients who are likely to experience voiding dysfunction after PVS surgery.'''
***Although urodynamic studies are useful in understanding the voiding dynamics of incontinent women, low detrusor pressure and Valsalva voiding preoperatively should not exclude patients from having an anti-incontinence procedure.
**'''<span style="color:#ff0000">Diagnosis and Evaluation'''
***'''<span style="color:#ff0000">History and Physical Exam'''
****'''Presentation of patients with obstruction by a PVS is variable'''
****'''<span style="color:#ff0000">Symptoms range from complete urinary retention, impaired detrusor, detrusor overactivity contractility and urgency incontinence to the less obvious irritative symptoms.'''
***** '''Persistent/increasing urgency incontinence and urgency (8-25%) are more common presenting symptoms in bladder outlet obstruction after a PVS procedure than frank retention'''
***** '''The incidence of permanent retention is usually ≤ 5%;''' the majority of patients who require clean intermittent catheterization after PVS placement had a neurogenic bladder preoperatively
****'''<span style="color:#ff0000">Physical Exam'''
*****'''<span style="color:#ff0000">Pelvic organ prolapse'''
******'''A key factor in assessing voiding dysfunction is the presence of prolapse that was either uncorrected at time of surgery or that occurred postoperatively.''' Prolapse of sufficient size may kink or angulate and externally compress the urethra. '''After surgery, apical, anterior, and posterior prolapse must be ruled out as a cause of the urethral obstruction.'''
*** '''<span style="color:#ff0000">Cystoscopy'''
****Useful to rule out bladder pathology, sling perforation, and a hypersuspended urethra
*** '''Urodynamics'''
****'''The most important criterion for a sling incision or urethrolysis remains the temporal relationship between the symptoms and the surgical procedure. Urodynamic studies are essential in these cases to diagnose and make an appropriate treatment plan.'''
** '''<span style="color:#ff0000">Management'''
*** '''See 2017 AUA Female SUI Guidelines Notes'''
*** '''<span style="color:#ff0000">Although transient urinary retention is common, most patients return to spontaneous voiding within the first 10 days'''
****Obstruction after an autologous PVS procedure usually improves or resolves with time
*** '''<span style="color:#ff0000">If symptoms persist, loosening the sling in the operating room can be attempted In the first 6 weeks after autologous PVS surgery.'''
****Technique
*****Using spinal or general anesthesia, this is done by first inserting a cystoscope into the bladder and then gently applying caudal pressure to the urethra
****'''This procedure is not advised with synthetic slings'''
*** '''<span style="color:#ff0000">After 6 weeks or when conservative measures fail, a sling incision or formal urethrolysis is indicated'''
**** '''Sling incision has comparable success rates and shorter operative time and less morbidity than formal urethrolysis'''
****Recurrent SUI after formal urethrolysis is reported as 0-19% and 34% after sling incision
****Urethrolysis can be performed by a retropubic, transvaginal, or suprameatal approach
**** Reported success rates of the surgical management of bladder outlet obstruction after a PVS procedure are 65-93%
****There are no preoperative or urodynamic parameters that consistently predict success or failure of urethrolysis.
*** '''Failure of urethrolysis'''
****'''Causes'''
*****'''Persistent or recurrent obstruction'''
*****'''Detrusor overactivity'''
*****'''Impaired detrusor contractility'''
*****'''Learned voiding dysfunction'''
****'''Management'''
*****'''The most common reason for failure is likely insufficient dissection and lysis of the urethra. This supports the use of repeat urethrolysis in the face of initial failure or in cases wherein the aggressiveness of the initial dissection is unknown.'''
*** '''Refractory storage symptoms after urethrolysis'''
****OAB symptoms are refractory in 50% of affected patients after urethrolysis and contribute to a significant portion of the reported failures.
****Can be challenging to treat.
*****In addition to anticholinergics, SNM should be considered as an option for de novo or refractory urgency and urgency incontinence after urethrolysis.
* '''Non-urologic Complications of PVS'''
** '''Most commonly pulmonary, cardiovascular, neurologic, and gastrointestinal (bowel injury)'''
=====Burch colposuspension=====
*Largely replaced by MUS
**Several RCTs showed essentially equivalent outcomes with the Burch colposuspension vs. TVT
*Likely inferior to pubovaginal fascial sling
*'''<span style="color:#ff0000">Indications (2):</span>'''
*#'''<span style="color:#ff0000">Patient preference to avoid mesh and avoid the morbidity of fascial harvest</span>'''
*#'''<span style="color:#ff0000">Undergoing a simultaneous abdominal procedure, such as open or minimally invasive hysterectomy</span>'''


== Questions ==
== Questions ==
Line 407: Line 665:


* Wein AJ, Kavoussi LR, Partin AW, Peters CA (eds): CAMPBELL-WALSH UROLOGY, ed 11. Philadelphia, Elsevier, 2015, chap 84
* Wein AJ, Kavoussi LR, Partin AW, Peters CA (eds): CAMPBELL-WALSH UROLOGY, ed 11. Philadelphia, Elsevier, 2015, chap 84
*[https://pubmed.ncbi.nlm.nih.gov/37096580/ Kobashi, Kathleen C., et al. "Updates to surgical treatment of female stress urinary incontinence (SUI): AUA/SUFU guideline (2023)." ''The Journal of Urology'' 209.6 (2023): 1091-1098.]

Latest revision as of 09:46, 11 March 2024

See 2023 AUA Female SUI Guideline Notes

Definitions[edit | edit source]

  • Stress Urinary Incontinence (SUI): symptom of urinary leakage due to increased abdominal pressure, which can be caused by activities such as sneezing, coughing, exercise, lifting, and position change.
  • Intrinsic sphincter deficiency (ISD): often defined as an abdominal leak point pressure <60 cm H20 or a maximal urethral closure pressure <20 cm H20, often in the face of minimal urethral mobility
    • The utility of urethral function assessment remains controversial
  • Urgency urinary incontinence (UUI): symptom of urinary leakage that occurs in conjunction with the feeling of urgency and a sudden desire to urinate that cannot be deferred.
  • Mixed urinary incontinence refers to a combination of SUI and UUI
  • Mesh located in the lower urinary tract is termed a perforation, and extrusion of mesh through the skin or vagina is termed exposure

Epidemiology[edit | edit source]

  • Prevalence of female SUI: up to 49%

Pathogenesis[edit | edit source]

  • Early theories regarding the cause of SUI were focused on the unequal transmission of pressure to the bladder and urethra, urethral hypermobility, and the abnormal inferior location of the urethra.
  • Later theories (integral theory) emphasized the importance of 3 separate components that support the proximal and midurethra:
    1. Pubourethral ligaments
    2. Suburethral vaginal hammock
    3. Pubococcygeus muscle
    • Injury to any of these components from surgery, parturition, aging, or hormonal deprivation can lead to impaired midurethral function and subsequently SUI
  • Intrinsic sphincter deficiency (ISD) is the primary underlying cause of SUI for women, with hypermobility being a secondary finding i.e. all women with SUI have some component of ISD, not all have hypermobility
    • Urethral hypermobility is a symptom of damage to the normal supporting structures of the urethra and not a cause of SUI
    • Maximum urethral closure pressures (MUCPs) < 20 cm H2O or abdominal leak point pressure (ALPP) < 60 cm H2O are indicative of ISD
      • In women who are continent, MUCP occurs at the midurethra
      • ALPP > 90 cm H2O signifies no or very little ISD

Differential Diagnosis of SUI[edit | edit source]

  • Options (7)
    1. Overflow incontinence
    2. Detrusor overactivity incontinence
    3. Low bladder compliance
    4. Stress-induced detrusor overactivity
    5. Diverticulum
    6. Urinary fistula
    7. Ectopic ureter

Diagnosis and Evaluation of Patients Wanting Surgery for SUI[edit | edit source]

Recommended Investigations[edit | edit source]

Mandatory (4):[edit | edit source]

  1. History (including assessment of bother) and Physical Exam
  2. Objective demonstration of stress urinary incontinence
  3. Urinalysis
  4. PVR

History and Physical Exam[edit | edit source]

History[edit | edit source]

  • Characterize incontinence (stress, urgency, mixed, continuous, without sensory awareness)
    • Presence of coughing, sneezing, lifting, walking, or running as initiators of incontinence increases the likelihood of SUI as the cause of urinary leakage
    • Chronicity of symptoms
    • Frequency, bother, and severity of incontinence episodes. Pad or protection use.
      • An assessment of bother is paramount to the decision to operate in the index patient.
  • Associated
    • Urinary tract symptoms (e.g., urgency, frequency, nocturia, dysuria, hematuria, slow flow, hesitancy, incomplete emptying)
    • Pelvic symptoms (e.g., pelvic pain, pressure, bulging, dyspareunia)
    • GI symptoms (e.g., constipation, diarrhea, splinting to defecate)
  • Menopausal status
  • Obstetric history (e.g., gravity, parity, method of delivery)
  • Previous pelvic surgeries
  • Past medical history (e.g., hypertension, diabetes, history of pelvic radiation)
  • Current and past medications
  • Fluid, alcohol, and caffeine intake
  • Previous treatments for incontinence (e.g., behavioral therapy, Kegel exercises/pelvic floor muscle training, pharmacotherapy, surgery)
  • Patient’s expectations of treatment (patient-centered goals)
  • History alone, while helpful, does not definitively diagnose SUI in women

Physical exam (6):[edit | edit source]

  1. Stress test (supine and/or standing) with comfortably full bladder
  2. Focused abdominal examination
  3. Urethral mobility (Q-tip test or other method)
  4. Pelvic prolapse (any method)
  5. Vaginal atrophy/estrogenization status
  6. Focused neurologic examination

Objective demonstration of SUI[edit | edit source]

  • Stress test (supine and/or standing) with comfortably full bladder
    • Considered positive if involuntary urine loss from the urethral meatus is witnessed coincident with increased abdominal pressure
      • A positive stress test had a high sensitivity and specificity for detecting SUI
    • If leakage is not witnessed in the supine position, the test may be repeated in the standing position to facilitate the diagnosis

Urinalysis[edit | edit source]

  • Screen for abnormalities (microscopic hematuria, pyuria, etc.), which may prompt further investigations and reveal underlying cause (bladder tumour, for example) of incontinence

Post-void Residual (PVR)[edit | edit source]

  • May prompt further investigations and reveal underlying cause (overflow, for example) of incontinence

Other Tests[edit | edit source]

Questionnaires[edit | edit source]
  • Overall, low strength of evidence due to limited number of studies for each questionnaire
Q-tip test[edit | edit source]
  • Positive test is unlikely to aid in the diagnosis of SUI; SUI may exist without urethral hypermobility and vice versa.
  • Can provide potentially useful information regarding the degree of urethral mobility
Pad test[edit | edit source]
  • May confirm the presence of incontinence but does not distinguish the specific type

Additional Evaluations[edit | edit source]

  • Should be considered for patients with the following conditions (8):
    1. Neurogenic lower urinary tract dysfunction (known or suspected)
    2. Inability to demonstrate stress urinary incontinence
    3. Inability to make definitive diagnosis based on symptoms and initial evaluation
    4. Elevated post-void residual per clinician judgment
    5. Evidence of significant voiding dysfunction
    6. Urgency-predominant mixed urinary incontinence
    7. Abnormal urinalysis, such as unexplained hematuria or pyuria
    8. High grade pelvic organ prolapse (POP-Q stage 3 or higher) if SUI not demonstrated by pelvic organ prolapse reduction
  • May be performed in patients with:**Concomitant overactive bladder symptoms
    • Failure of prior anti-incontinence surgery
    • Prior pelvic prolapse surgery

Cystoscopy[edit | edit source]

  • Should not be performed in index patients for the evaluation of SUI
  • Indications (3):
    1. Suspected bladder pathology based on history or concerning findings on physical exam or urinalysis (e.g. microhematuria)
    2. Structural lower urinary tract abnormality
    3. Patients undergoing certain surgical procedures (e.g., midurethral (MUS) or pubovaginal fascial (PVS) slings) to confirm the integrity of the lower urinary tract and the absence of foreign body
    4. History of prior anti-incontinence surgery or pelvic floor reconstruction, particularly if mesh or suture perforation is suspected
      • Perforation should be suspected with new onset of lower urinary tract symptoms, hematuria, or recurrent UTI

UDS[edit | edit source]

  • May be omitted for the index patient desiring treatment when SUI is clearly demonstrated
  • May be performed at the urologist’s discretion in certain non-index patients, including but not limited to (8):
    1. Neurogenic lower urinary tract dysfunction
    2. Unconfirmed SUI
    3. Mismatch between subjective and objective measures
    4. Elevated PVR per clinician judgment
    5. Significant voiding dysfunction
    6. Significant urgency, UUI, overactive bladder (OAB)
    7. History of prior pelvic organ prolapse surgery
    8. History of prior anti-incontinence surgery
  • If significant prolapse is present, UDS should be performed with and without a pessary
  • In patients with DO, treatment options other than sling surgery should be considered because stress-induced DO may be difficult to treat with a sling alone.
  • Abnormally small bladder capacity and decreased compliance may also negatively affect the outcomes of sling surgery, and these factors should also be considered.

Management[edit | edit source]

Options[edit | edit source]

  • Observation
  • Non-surgical interventions (4):
    1. Urethral plugs
    2. Vaginal inserts
    3. Continence pessary
    4. Pelvic floor muscle training (± biofeedback)
  • Surgical intervention
    • Options (4):
      1. Periurethral bulking agents
      2. Midurethral sling (MUS)
      3. Autologous pubovaginal sling (PVS)
      4. Burch culposuspension
  • Physicians should not offer stem cell therapy for SUI outside of investigative protocols

Observation[edit | edit source]

Indications (2)[edit | edit source]
  1. Patient expresses minimal subjective bother due to the SUI
    • The degree of bother should be considered when considering treatment
  2. Not candidate for other forms of therapy

Non-surgical interventions[edit | edit source]

  • Options (4):
  1. Urethral plugs
  2. Vaginal inserts
  3. Continence pessary
  4. Pelvic floor muscle training (± biofeedback)
Pelvic floor physical therapy[edit | edit source]
  • Patient must be willing and able to commit to regularly and consistently performing pelvic floor training for this to be successful.

Surgical intervention[edit | edit source]

  • In most cases of incontinence, surgery should not be considered until more conservative management has failed.
  • Options (4):
  1. Bulking agents
  2. Synthetic midurethral slings (MUS)
  3. Autologous pubovaginal sling (PVS)
  4. Burch culposuspension
Bulking agents[edit | edit source]
  • Trade name: Bulkamid
    • FDA approved in 2020§
  • Little long-term data
  • Indications (2):
    1. Patients who wish to avoid more invasive surgical management or are concerned with the lengthier recovery time after surgery
    2. Patients who experience insufficient improvement following a previous anti-incontinence procedure.
  • Disadvantage (1)
    1. Patients should be counseled on the expected need for repeat injections
  • Adverse events§
    • Pain at implantation (13%)
    • Acute urinary retention (6%)
    • Urinary tract infection (4%)
    • Blood in urine (2%)
Midurethral Slings (MUS)[edit | edit source]
  • Most studied surgical treatment for female SUI
  • Other than bulking agents, MUS is also the least invasive surgical options to treat SUI
Classification (3):[edit | edit source]
  1. Retropubic MUS (RMUS, e.g. TVT-R); top-down or bottom-up
  2. Transobturator MUS (TMUS e.g. TVT-O); inside-out or outside-in
    • TVT without specification refers to TVT-R
  3. Single incision sling (SIS)/adjustable sling types
Mechanism of Action (2)[edit | edit source]
  1. Restricts movement of the posterior urethral wall above the sling , directing its motion in an anteroinferior or anterior direction.
    • Patients without urethral hypermobility do not respond as well to MUS surgery
      • Lack of urethral mobility is an indication that the patient has a fixed urethra and ISD
  2. Narrows (compresses) the urethral lumen due to inward movement of the posterior urethral wall after placement of a MUS
Anatomy[edit | edit source]
  • Anatomy of the Retropubic Midurethral Sling
    • The left and right dorsal nerves of the clitoris (DNCs) run along the inferior surface of the ischiopubic rami and cross under the pubic bone approximately 1.4 cm from the midline
    • The obturator vessels are the closest major vascular structures to a retropubic MUS
  • Anatomy of the Transobturator Midurethral Sling
    • The transobturator technique is unique because (when done correctly) it avoids entry into the true pelvis and the levator group.
      • Muscle’s traversed by this technique include the obturator internus muscle, obturator membrane, and obturator externus muscle as it goes through the obturator foramen, and the adductor muscles (adductor magnus, adductor brevis, and gracilis), lateral to the obturator foramen
        • Errant sling placement through the adductor longus tendon can result in substantial pain.
    • The obturator vessels are lateral and superior to the area of insertion of the device.
    • The dorsal nerve of the clitoris is separated from the trajectory of the device by ≥1-2 cm
Synthetic Sling Materials[edit | edit source]
  • The initial MUSs were made of materials with smaller pore sizes
  • Currently, a soft, loosely woven, polypropylene monofilament mesh with a pore size > 75 μm is the most commonly used material
    • Trend in the contemporary literature toward the use of macroporous polypropylene slings
      • The increased pore size of these materials allows for: excellent tissue ingrowth, promotes integration with the surrounding host tissues, and decreases encapsulation and infection
Contraindications[edit | edit source]
  • Should not utilize a synthetic MUS in patients undergoing concomitant (3):
    1. Urethral diverticulectomy
    2. Repair of urethrovaginal fistula
    3. Urethral mesh excision
    • Mesh placed in close proximity to a concurrent urethral incision can theoretically affect wound healing, potentially resulting in mesh perforation.
    • Instead, an anti-incontinence procedure that does not involve placement of synthetic material suburethrally, or use of a biologic material, preferably autologous fascia, should be considered.
  • Consider avoiding the use of mesh in patients undergoing SUI surgery who are at risk for poor wound healing, including
    1. Following radiation therapy
    2. Presence of significant scarring
    3. Poor tissue quality
    4. Long-term steroid use
    5. Impaired collagen associated with systemic autoimmune disorders, such as visceral Sjogren’s disease or systemic lupus erythematosus
    6. Immune suppression
Technique[edit | edit source]
  • Should be placed loosely at the midurethra because its function is not primarily related to compression.
    • A loosely placed MUS combined with a mobile urethra may allow the sling to compress the urethra during times of Valsalva and stress while remaining nonobstructive when the urethra is at rest.
      • For single-incision MUSs, tension should be tighter than the classic retropubic or transobturator MUS surgeries to achieve the same result
        • Unlike retropubic and transobturator MUSs, for single-incision MUSs it appears that restriction of urethral mobility after surgery is associated with a better outcome
  • Cystoscopy is performed to exclude trocar penetration of the lower urinary tract.
    • If bladder perforation is noted, the trocar is withdrawn and passed once more with an effort to avoid further perforation
Outcomes[edit | edit source]
  • Predominantly SUI
    • Retropubic vs. Trans-obturator synthetic midurethral sling
      • Similar outcomes, regardless of urethral function
      • Long-term comparisons are relatively lacking, however, data from increasing follow up appear to be demonstrating a lack of durability of Transobturator MUS, compared to Retropubic MUS
    • Single-Incision MUS in Patients with Predominantly SUI
      • There are fewer data available regarding the safety and efficacy of this new generation of slings compared with the retropubic and transobturator MUSs.
      • Single-incision slings have decreasing efficacy with longer follow-up.
  • MUI
    • Results with mixed incontinence are acceptable compared with other types of interventions for urinary incontinence but are less than those obtained in pure SUI.
  • Intrinsic Sphincteric Deficiency
    • The success of MUSs is lower in patients with ISD/fixed urethra (no urethral mobility) and low leak point pressures
      • Fixed urethras have poor outcomes after MUS surgery regardless of leak point pressure
      • Retropubic MUS are more effective than transobturator MUS in patients with ISD
    • Low leak point pressures are not necessarily a contraindication to retropubic MUS surgery
    • MUS is beneficial in the management of SUI in patients with ISD as long as there is preoperative urethral mobility; urethral mobility before MUS procedures has been shown to be predictive of success; the more the proximal urethra moves during a Valsalva maneuver, the better the cure rate for incontinence
  • Efficacy and safety of MUSs are not compromised in those undergoing concomitant vaginal surgery, the elderly, or the obese.
    • Pelvic Organ Prolapse
      • Results suggest that the MUS can be added to prolapse surgery with minimal morbidity.
        • The Cochrane Incontinence Group reviewed 22 randomized trials of surgical prolapse repair including 2368 women. They concluded that the addition of a retropubic MUS to endopelvic fascial plication, Burch colposuspension, and abdominal sacrocolpopexy may reduce the incidence of postoperative SUI, but issues of cost and associated adverse effects were unclear
        • Women who underwent prolapse repair at the time of the sling surgery were significantly more likely to be diagnosed with postoperative outlet obstruction (9.4% vs. 5.5%, P < .007), but less likely to undergo a repeat procedure for stress incontinence or reoperation for prolapse within 1 year after sling surgery.
        • When MUSs are placed for urodynamic or occult SUI at time of prolapse repair, the risk of intervention because of obstruction is equivalent to the risk of intervention for SUI if no MUS was placed (8.5% and 8.3%, respectively)
      • MUSs placed with either transvaginal or laparoscopic-assisted vaginal hysterectomy and anterior or posterior colporrhaphy have been shown to have success rates similar to those in published series of MUS surgery alone. Complication rates are also in accordance with other MUS series
    • Elderly Patients
      • Cure rates in older women with urethral hypermobility are comparable to those in younger women.
        • Elderly women should not be excluded from potentially curative MUS surgery based on their age alone.
        • Mixed urinary incontinence resolution rates are similar to those of the younger population
        • Possibly because of preoperative factors such as MUI or even decreased urethral hypermobility, the rate of persistent SUI after retropubic or transobturator MUS procedures appears higher in the elderly population.
      • Complication rates vary, with some studies citing a higher rate of age-related morbidities but no apparent increase in intraoperative complications.
        • Elderly patients experience higher rates of postoperative de novo urgency and urgency incontinence associated with any sling material, including MUS
        • Postoperative retention occurs to a similar degree as in younger patients
    • Obese Patients
      • Controversial whether obesity affects surgical outcome with MUSs
      • Overall, the rate of complications appears to be similar in obese versus non-obese patients undergoing MUS surgery.
        • Higher rate of bladder trocar injury in non-obese patients
  • Recurrent Stress Urinary Incontinence
    • As salvage procedures, MUS have overall efficacy similar to their use in primary implantation procedures.
    • The procedure can be performed in the same way as it is performed for primary SUI.
    • The complication rate is similar to that of retropubic MUSs done for primary SUI, but the risk of bladder perforation appears to be higher in females who have had ≥1 prior retropubic suspensions.
    • As is the case with primary surgery, the failure rate is higher in females with immobile urethras.
    • No significant difference in subjective cure rates in patients after retropubic vs. transobturator MUS surgery for recurrent SUI.
Adverse events[edit | edit source]
  • Patient counselling
    • Permanent nature of these products since MUS surgery involves the implantation of a synthetic, prosthetic material
    • Unique and sometimes serious complications related to their use
    • Risk of transient and permanent voiding dysfunction after surgery, including postoperative difficulty emptying the bladder and de novo urgency and frequency
  • Overall, relatively low rates of complications (10)
    1. Bladder trocar injury: 2.7-3.8%; higher with retropubic
    2. Voiding dysfunction: 7.6%
    3. Wound healing problems: 1%
    4. Vaginal mesh exposure: 0.5-8.1%
    5. Mesh perforation of urethra (transobturator and retropubic): 0-0.6%
    6. Mesh perforation of bladder: 0.5-0.6%
    7. Sexual dysfunction
    8. Groin pain; higher with transobturator; conservative therapy with NSAIDs should resolve the majority of symptoms. If pain persists after 6-8 weeks, consider referral to a pain clinic for trigger point injections and physical therapy.
    9. Bleeding, vascular injury; higher with retropubic
    10. Infection, bowel perforation, and death
  • Bladder Trocar Injury
    • Rate of trocar injury is higher with retropubic vs. trans-obturator MUS
      • The rate of bladder or urethral trocar injury:
        • Retropubic MUS surgery: 2.7-23.8%
        • Trans-obturator MUS surgery: 0-1.3%
    • Management
      • If occurs intra-operatively, remove trocar and pass trocar again more laterally and leave foley catheter in 3-7 days.
        • Trocar injury is generally thought of as a benign condition.
  • Vaginal Mesh Exposure
    • Most cases manifest within a few weeks to a few months after the MUS procedure
    • Risk factors (5):
      1. Diabetes
      2. History of smoking
      3. Older age
      4. >2 cm vaginal incision length
      5. Previous vaginal surgery
    • Diagnosis and Evaluation
      • History and Physical Exam
        • Signs and Symptoms (6):
          1. Vaginal discharge (with variable constituents and different amounts of blood and inflammatory components)
          2. Palpable rough surface in the vagina
          3. Sexual discomfort (including partner related)
          4. Pelvic pain
          5. Inguinal discomfort
          6. LUTS (urgency, frequency, persistent incontinence, hematuria)
    • Management
      • The management of this complication is not standardized, composition of mesh is particularly important in the event of mesh exposure
      • Options (2)
        • Conservative management with conjugated estrogen and possibly antibiotic creams
          • Option in well-selected patients who are relatively asymptomatic and have small-caliber exposures (<1 cm)
        • Excision should be reserved for failure of conservative therapy or when local symptoms mitigate against observational management (e.g., bothersome dyspareunia).
          • Limited excision and trimming with vaginal closure can be attempted.
            • Even with partial excision of the mesh, continence is maintained in the majority of patients.
          • If these options fail, excision of most of the mesh from a transvaginal approach should be pursued in most cases.
            • Operative management typically involves excision of the exposed mesh, thorough irrigation with antibiotic solution, and closure of vaginal flaps. Good results have also been observed in selected patients with vaginal advancement flaps and suture approximation of the debrided vaginal mucosa over the exposed mesh.
  • Mesh Perforation of the Urethra
    • Diagnosis and Evaluation
      • History and Physical Exam
        • History
          • Presenting symptoms are variable
          • Voiding dysfunction is predominant
    • Management
      • Management of this complication is extremely challenging
      • Observation should never be considered when there is urethral perforation
      • Options (2)
        1. Endoscopic management of small areas of mesh perforation
        2. Transvaginal surgical excision
          • For slings that perforate into the urethra an inverted-U incision is best because this allows for exposure of the proximal urethra, bladder neck, and endopelvic fascia as well as providing a vaginal epithelial flap that avoids overlapping suture lines
          • An autologous fascial sling or a Martius labial fat pad graft can be used for repair, at the discretion of the surgeon.
            • An autologous fascial sling can be placed at the time of surgery to augment the repair or in a delayed fashion to treat recurrent SUI.
  • Mesh Perforation of the Bladder
    • Pathogenesis
      • Most likely the result of an unrecognized cystotomy or placement of the mesh within the urinary bladder at the time of surgery.
    • Diagnosis and Evaluation
      • History and Physical Exam
        • Signs and Symptoms
          1. Lower abdominal pain
          2. Intermittent gross hematuria
          3. Recurrent UTI
          4. Urgency
          5. Frequency
          6. Dysuria
          7. Urinary incontinence
    • Management
      • Observation should never be considered when there is intravesical perforation
      • Options (2)
        1. If small area of mesh perforation: endoscopic management (excision with scissors or ablation with the holmium laser) is an appropriate initial step if complete excision of all exposed material can be achieved
        2. If endoscopic excision fails or as initial treatment for large areas of mesh perforation, transvaginal or retropubic excision
          1. Reconstruction should involve nonoverlapping suture lines and interposition of tissue such as a labial fat pad or greater omentum.
          2. An autologous fascial sling can be placed at the time of surgery to augment the repair or in a delayed fashion to treat recurrent SUI.
  • Pain and Infection after MUS Surgery
    • Groin and suprapubic pain are potential problems after MUS placement.
      • Thigh and groin pain appear to be more commonly associated with the transobturator approach
    • Severe infection is a rare complication after MUS surgery
      • Diagnosis of this complication is variable and can take as long as several years
      • Obesity, diabetes, and hypertension are associated with fasciitis after pelvic surgery.
    • Management of MUS Severe Infection or Pain
      • In most cases, postoperative groin or leg pain after MUS surgery can be managed with NSAIDs, rest, and physical therapy.
        • Most groin pain resolves after postoperative day 2
          • Pain persists longer after the transobturator midurethral slings
      • In instances of chronic mesh pain and severe infection when nonoperative therapy has failed, it may be necessary to attempt a complete mesh excision from both sides of the bone.
      • For the complete excision of transobturator mesh we typically consult an orthopedic surgeon to aid with lateral dissection of the sling.
  • Voiding Dysfunction after MUS Surgery
    • Epidemiology
      • De novo urgency occurs with postoperative voiding dysfunction in as many as 12% of patients
      • Voiding dysfunction after MUS is substantially less than with PVS (bladder neck) slings
    • Usually transient
      • Long-term urinary retention and obstructive voiding dysfunction are rare after the MUS procedure
    • Pathogenesis
      • Typically the result of obstruction
        • From the sling being placed too tightly or in the wrong location (too proximally)
        • Associated with pelvic organ prolapse (unrecognized preoperatively or de novo)
          • Some patients may have voiding dysfunction without evidence of obstruction
    • Risk factors
      • Type of MUS
        • In general, similar rates of de novo urgency and perioperative urinary retention among the different types of MUSs
        • However, an RCT found significantly higher rate of voiding dysfunction necessitating surgery (or permanent catheter) after a retropubic MUS compared with a transobturator sling procedure (3% vs. 0%)§
        • In addition, the rate of urinary retention (catheter for longer than 6 weeks) was also higher in the retropubic MUS group (3.7% vs. 0.7%).
    • Diagnosis and Evaluation
      • History and Physical Exam
        • History
          • Most common symptoms of obstruction are
            • Inability to void (urinary retention)
            • Incomplete emptying
            • De novo urgency and frequency
          • Over several weeks to a month, the storage symptoms (urgency and frequency) and pain become more prevalent as the bladder attempts to adjust to the obstruction.
        • Physical Exam
          • Pelvic organ prolapse
            • A key factor in assessing voiding dysfunction is the presence of prolapse that was either uncorrected at time of surgery or that occurred postoperatively. Prolapse of sufficient size may kink or angulate and externally compress the urethra. After surgery, apical, anterior, and posterior prolapse must be ruled out as a cause of the urethral obstruction
      • Cystoscopy
        • Useful to rule out bladder pathology, urethral mesh perforation, and a hypersuspended bladder neck
      • The optimal evaluation for patients with postoperative voiding dysfunction is poorly defined in the literature. The decision to perform urethrolysis is usually based on a clear temporal relationship between onset of symptoms and the surgical procedure.
    • Management
      • Urinary obstruction after MUS surgery is usually transient and can be managed with short-term intermittent catheterization
      • If persistently elevated residual urine, bothersome symptoms refractory to conservative management, and within first 3 months of surgery, then perform transvaginal sling release
        • Cutting the MUS in the midline through a single vertical vaginal incision using minimal dissection is the preferred method to manage persistent voiding dysfunction that results from an obstructive sling within the first 3 months after surgery
          • Transvaginal sling release consistently provide resolution of symptoms with maintenance of continence in the majority of patients
            • The entire sling does not need to be excised; majority of patients maintain continence with single incision
        • Timing
          • The exact timing of sling incision is variable; however, most recommend waiting at least 2 weeks; the sling should be incised within 4 weeks of surgery
      • If after 3 months, perform a more formal sling excision and urethrolysis
        • Sling may be fixed along its entire course, and midline sling incision may not achieve enough sling relaxation to resolve voiding dysfunction
      • Urethral dilation is of limited usefulness and, if used too aggressively, may be detrimental.
  • Sexual Dysfunction after MUS
    • Contradictory evidence in the literature that MUS surgery improves and worsens sexual function
      • Some attribute improved sexual function after MUS surgery to a significant decrease in coital incontinence
      • The rate of de novo dyspareunia after MUS surgery is between 3-14%; sling removal can improve dyspareunia
  • Other Complications after MUS Surgery
    • Infection, bleeding, vascular injury, bowel perforation, and death.
      • Risk of major vascular injury higher with retropubic vs. transobturator
      • The majority of postoperative hematomas resolve without intervention
    • UTI is the most common and easily treatable complication of MUS surgery.
Regulatory and Legal Issues Related to Sling Mesh Complications[edit | edit source]
  • See AUA/CUA Mesh Position Statements
  • The success of MUSs for incontinence led to the development of mesh products for pelvic organ prolapse repair. However, while the use of mesh during pelvic organ prolapse repairs has increased, so too have complications related to its use.
  • Synthetic mesh for pelvic organ prolapse
    • In October 2008, the FDA released a public health notification (PHN) alerting the public about “rare” complications and problems related to transvaginal mesh products used for pelvic organ prolapse.
    • In 2011, the FDA modified this alert by removing the term “rare” and stating that surgical mesh for pelvic organ prolapse repair does not conclusively improve outcomes over traditional nonmesh or native tissue repairs and is associated with unique potentially serious adverse outcomes
  • Synthetic mesh for midurethral slings
    • FDA advisory panel deemed existing MUS products “safe and effective”
    • Even though the FDA has determined that existing MUS products are safe and effective, this unfortunately does not prevent MUSs from being caught up in the fervor of litigation related to products used for pelvic organ prolapse repair.
    • Single-incision sling manufacturers are required to perform 522 postmarket surveillance studies
Pubovaginal Slings (PVS)[edit | edit source]
  • Indicated for treatment of incontinence associated with
    • Deficiency in a portion of the midurethral complex
    • Hypermobility
    • ISD
    • MUI
    • Concomitant cystoceles
    • Urethral diverticula
    • Neurologic conditions
Anatomy and Mechanics of a PVS[edit | edit source]
  • Positioned at the bladder neck (in contrast to MUS which is placed in the midurethra) to provide urethral compression without obstruction during times of increased intraabdominal pressure.
    • Pubovaginal slings are placed under mild tension at the bladder neck to reestablish the suburethral hammock and are able to improve SUI by providing a layer of tissue that compresses the urethra during times of increased intra-abdominal pressure.
    • Unlike the PVS, the MUS should be placed loosely at the midportion of the urethra to prevent movement of the posterior urethral wall
  • The current concept of PVS comes from using a shorter free graft of rectus fascia whose tension could be adjusted
Pubovaginal Sling Materials[edit | edit source]
  • Options (4):
    1. Autologous
    2. Allograft
    3. Xenograft
    4. Synthetic
      • Although there is complete biocompatibility of the autologous sling and negligible urethral perforation, biologic graft and synthetic prosthetic materials have been increasingly used to decrease operative time, morbidity, pain, and hospital stay
  • Autologous fascia PVS
    • Gold standard for management of ALL forms of SUI
    • Advantages compared to alternative sling materials (2):
      1. Minimal tissue inflammation
      2. Negligible risk of urethral erosion
    • Disadvantages compared to alternative sling materials (4):
      • Increased operative time, hospital stay
      • Postoperative pain
      • Risk of suprapubic wound seroma
      • Risk of incisional hernia
    • Most commonly used autologous materials (2):
      1. Rectus abdominis fascia harvested from the abdominal wall
        • Most commonly used
      2. Fascia lata harvested from the lateral thigh
        • Fascia lata is the preferred autologous material for PVSs in patients with a history of prior ventral hernia repair; unlike rectus facia, the recovery time is less and there is no risk of future abdominal hernia formation.
        • Disadvantages of fascia lata compared to rectus abdominis:
          1. Requires repositioning of the patient
          2. Increased operative time
          3. Operating in an area unfamiliar to most pelvic surgeons
      3. Rectus abdominis fascia vs. fascia lata PVS: similar improvement of SUI
  • Allograft PVS
    • Were introduced in an effort to reduce overall morbidity, operative time, and pain related to graft procurement
    • Currently derived from either cadaveric fascia lata or acellular human dermis
    • Allografts from cadavers raise the concern of potentially transmitting illnesses such as HIV, hepatitis, and Creutzfeldt-Jakob prion disease
      • The estimated risk of HIV transmission from an allograft is 1 in 1,667,600.
      • The theoretical risk of developing Creutzfeldt-Jakob disease from a non-neural allograft is 1 in 3.5 million.
    • Tissue-processing techniques for allografts may disrupt the microstructure and affect their strength properties
      • Maximum load to failure, maximum load/graft width, and stiffness are significantly lower for the allograft freeze-dried fascia lata group compared with the autologous, solvent-dehydrated, and dermal graft groups
  • Xenograft PVS
    • The forms of xenograft used are porcine dermis, porcine small intestinal submucosa, and bovine pericardium
    • Less tensile strength than allograft in situ and highest propensity to encapsulate
  • Synthetic PVS
    • The most commonly used synthetic material for PVSs is polypropylene mesh.
    • Advantages:
      • Almost unlimited supply of artificial graft material in various sizes and shapes, consistency in quality, elimination of harvest site complications, and decreased operative time.
      • More uniform, consistent, and durable compared with biologic grafts
      • Sterile, biocompatible, and noncarcinogenic
      • Lowest amount of degradation or disruption and the highest amount of fibroblast ingrowth and tissue ingrowth into the specimen
    • Disadvantages
      • Significant inflammatory and foreign body reactions
      • Higher rates of graft infection, urinary tract perforation, and vaginal exposure
        • No longer used to due risk of complications

Patient counselling

  • If a synthetic prosthetic or biologic graft material is being used, surgeons should thoroughly counsel their patients about the permanent nature of these products and the unique and sometimes serious complications related to their use.
  • Risk of transient and permanent voiding dysfunction after surgery, including postoperative difficulty emptying the bladder and de novo urgency and frequency
Technique[edit | edit source]
  • Graft Harvest for Autologous Pubovaginal Sling
    • Closure of the rectus fascia without tension is sometimes problematic. To prevent this difficulty, it is important to maintain a distance of ≥2 cm from the pubic symphysis
    • If undermining the fascial edges does not adequately mobilize the fascia, than interposition of a segment of synthetic mesh or graft may be necessary.
  • Pubovaginal Sling Placement and Fixation
    • The bladder must be completely drained before passage of the Stamey needles to avoid inadvertent bladder injury
    • Perform a cystoscopy after trocar passage to ensure integrity of the bladder and at the time of sling tensioning to visualize the bladder neck
    • A sling should never be tensioned before the weighted speculum is removed and the vaginal incision is closed.
      • Tensioning before this may result in failure of the procedure due to too much or too little tension. The abdominal incision is closed after the sling is tensioned.
    • No suture fixation to the underlying periurethral fascia is necessary to anchor the sling  
Outcomes[edit | edit source]
  • Predominantly SUI
    • No risk factors that consistently predict outcomes
    • PVS are particularly helpful in treating ISD (in contrast to midurethral sling)
    • The reported cure rate of PVS surgery for recurrent SUI is excellent
    • Autologous PVS
      • Continence rate after PVS ranges from 61-97%; wide range due to variation in outcome definition
      • Postoperative de novo or urgency incontinence rates range from 2-21%
        • Most common reason for failure/patient dissatisfaction relates to urgency symptoms and urgency incontinence at follow-up
    • Allograft PVS
      • Limited outcome data, and the efficacy and durability of these slings are questionable
      • Previously reported failures coupled with the consistent success and rapid adoption of synthetic MUSs has led to abandonment of all types of cadaveric allograft at most centers.
    • Xenograft PVS
      • Because of the morbidity of autologous fascial harvest, high failure rates of allograft materials, and high exposure and perforation rates with synthetic PVSs, xenografts are an attractive option.
      • In general, they are associated with a low rate of infection, exposure, and perforation owing to their incorporation into host tissue cure rates comparable to those of the autologous sling.
      • In RCTs, porcine dermis was associated with significantly inferior long-term cure rates compared with the autologous PVS
  • Autologous PVS for Mixed Urinary Incontinence
    • Overall, PVS remains an effective treatment option for MUI with cure rates similar to those of simple SUI.
      • The treatment of patients with mixed urgency and SUI is complicated and often involves a combination of anticholinergic therapy and surgery
      • PVS is an effective treatment option for stress-induced DO with cure rates similar to those of simple DO.
      • Anti-incontinence surgery may cure or aggravate urgency symptoms or lead to de novo urgency. This aspect of anti-incontinence surgery is unpredictable and a major cause of patient dissatisfaction
      • The presence of residual urgency is similar to de novo urgency with a PVS.
  • Autologous PVS for Urethral Reconstruction
    • Autologous PVSs in the setting of urethral reconstruction (urethral fistula, urethral diverticulum, destroyed urethra) has excellent results when compared with other surgeries for incontinence
Adverse Events[edit | edit source]
  • Pubovaginal Sling Perforation and Exposure
    • Incidence is partially dependent on the composition of sling material
      • Synthetic slings perforate 15x more often into the urethra and are exposed 14x more often in the vagina than autologous, allograft, and xenograft slings.
      • Perforation or exposure of autologous pubovaginal slings is rare
        • Urethral perforation rate was 0.02% and the vaginal exposure rate was 0.007% in 1515 patients who received synthetic slings.
        • Urethral perforation incidence of 0.003% and a vaginal exposure incidence of 0.0001% in 1715 patients undergoing autologous and allograft sling procedures.
      • Because urinary tract perforation and vaginal exposure of synthetic PVSs are more common and associated with significant morbidity, synthetic material is no longer used for bladder neck slings
    • Diagnosis and Evaluation
      • Urethral perforations present at ≈9 months
      • History and Physical Exam
        • Presenting symptoms often include urinary retention, urgency, and mixed incontinence.
        • In addition, synthetic sling perforations and exposures are also associated with vaginal discharge, vaginal pain, suprapubic pain, and recurrent UTIs.
    • Management
      • Management of autologous and allograft PVS urethral perforation usually involves incision or excision of the part of the sling that has perforated and simple closure of the urethra
      • The incidence of recurrent SUI after synthetic PVS urethral perforation is 44-100%, and treatment often involves a second PVS
  • Voiding Dysfunction Secondary to Bladder Outlet Obstruction after PVS
    • PVS is associated with higher success rate but increased risk of post-operative voiding dysfunction compared to the Burch colposuspension
    • Risk Factors
      • No well-established risk factors for patients who are likely to experience voiding dysfunction after PVS surgery.
      • Although urodynamic studies are useful in understanding the voiding dynamics of incontinent women, low detrusor pressure and Valsalva voiding preoperatively should not exclude patients from having an anti-incontinence procedure.
    • Diagnosis and Evaluation
      • History and Physical Exam
        • Presentation of patients with obstruction by a PVS is variable
        • Symptoms range from complete urinary retention, impaired detrusor, detrusor overactivity contractility and urgency incontinence to the less obvious irritative symptoms.
          • Persistent/increasing urgency incontinence and urgency (8-25%) are more common presenting symptoms in bladder outlet obstruction after a PVS procedure than frank retention
          • The incidence of permanent retention is usually ≤ 5%; the majority of patients who require clean intermittent catheterization after PVS placement had a neurogenic bladder preoperatively
        • Physical Exam
          • Pelvic organ prolapse
            • A key factor in assessing voiding dysfunction is the presence of prolapse that was either uncorrected at time of surgery or that occurred postoperatively. Prolapse of sufficient size may kink or angulate and externally compress the urethra. After surgery, apical, anterior, and posterior prolapse must be ruled out as a cause of the urethral obstruction.
      • Cystoscopy
        • Useful to rule out bladder pathology, sling perforation, and a hypersuspended urethra
      • Urodynamics
        • The most important criterion for a sling incision or urethrolysis remains the temporal relationship between the symptoms and the surgical procedure. Urodynamic studies are essential in these cases to diagnose and make an appropriate treatment plan.
    • Management
      • See 2017 AUA Female SUI Guidelines Notes
      • Although transient urinary retention is common, most patients return to spontaneous voiding within the first 10 days
        • Obstruction after an autologous PVS procedure usually improves or resolves with time
      • If symptoms persist, loosening the sling in the operating room can be attempted In the first 6 weeks after autologous PVS surgery.
        • Technique
          • Using spinal or general anesthesia, this is done by first inserting a cystoscope into the bladder and then gently applying caudal pressure to the urethra
        • This procedure is not advised with synthetic slings
      • After 6 weeks or when conservative measures fail, a sling incision or formal urethrolysis is indicated
        • Sling incision has comparable success rates and shorter operative time and less morbidity than formal urethrolysis
        • Recurrent SUI after formal urethrolysis is reported as 0-19% and 34% after sling incision
        • Urethrolysis can be performed by a retropubic, transvaginal, or suprameatal approach
        • Reported success rates of the surgical management of bladder outlet obstruction after a PVS procedure are 65-93%
        • There are no preoperative or urodynamic parameters that consistently predict success or failure of urethrolysis.
      • Failure of urethrolysis
        • Causes
          • Persistent or recurrent obstruction
          • Detrusor overactivity
          • Impaired detrusor contractility
          • Learned voiding dysfunction
        • Management
          • The most common reason for failure is likely insufficient dissection and lysis of the urethra. This supports the use of repeat urethrolysis in the face of initial failure or in cases wherein the aggressiveness of the initial dissection is unknown.
      • Refractory storage symptoms after urethrolysis
        • OAB symptoms are refractory in 50% of affected patients after urethrolysis and contribute to a significant portion of the reported failures.
        • Can be challenging to treat.
          • In addition to anticholinergics, SNM should be considered as an option for de novo or refractory urgency and urgency incontinence after urethrolysis.
  • Non-urologic Complications of PVS
    • Most commonly pulmonary, cardiovascular, neurologic, and gastrointestinal (bowel injury)
Burch colposuspension[edit | edit source]
  • Largely replaced by MUS
    • Several RCTs showed essentially equivalent outcomes with the Burch colposuspension vs. TVT
  • Likely inferior to pubovaginal fascial sling
  • Indications (2):
    1. Patient preference to avoid mesh and avoid the morbidity of fascial harvest
    2. Undergoing a simultaneous abdominal procedure, such as open or minimally invasive hysterectomy

Questions[edit | edit source]

  1. In the integral theory, what components support the proximal and midurethra
  2. What UDS values are suggestive of intrinsic sphincter deficiency?
  3. List 5 differences between MUS and PVS
  4. What are the most commonly used autologous materials in PVS?
  5. What are advantages vs. disadvantages of synthetic PVS?
  6. What were the main findings of the SISTEr trial?
  7. What are the typical presenting symptoms in a patient with obstruction following PVS?
  8. What is the surgical management of voiding dysfunction after PVS?
  9. What is the management of PVS perforation?
  10. What is the closest vascular structure related to the retropubic MUS approach?
  11. How do outcomes in the elderly differ from the young after MUS surgery?
  12. List complications associated with MUS
  13. What is the management of urinary retention after MUS?

Answers[edit | edit source]

  1. In the integral theory, what components support the proximal and midurethra
    1. Pubourethral ligaments
    2. Suburethral vaginal hammock
    3. Pubococcygeus muscle
  2. What UDS values are suggestive of intrinsic sphincter deficiency?
  3. List 5 differences between MUS and PVS
    • PVS: positioned at bladder neck, predominantly uses autologous material, complicated by donor graft morbidity, corrects ISD and hypermobility, can be used to correct SUI concomitant with urethral diverticular repair, higher risk of post-operative voiding dysfunction
    • MUS: positioned at midurethra, predominantly uses synthetic material, not complicated by donor morbidity, corrects hypermobility only, cannot be used to correct SUI concomitant with urethral diverticular repair, lower risk of post-operative voiding dysfunction
  4. What are the most commonly used autologous materials in PVS?
  5. What are advantages vs. disadvantages of synthetic PVS?
  6. What were the main findings of the SISTEr trial?
  7. What are the typical presenting symptoms in a patient with obstruction following PVS?
  8. What is the surgical management of voiding dysfunction after PVS?
  9. What is the management of PVS perforation?
  10. What is the closest vascular structure related to the retropubic MUS approach?
  11. How do outcomes in the elderly differ from the young after MUS surgery?
  12. List complications associated with MUS
  13. What is the management of urinary retention after MUS?

References[edit | edit source]